Está en la página 1de 48

w

U
o
z
,--

01 . Semiología urológica 03. Urolitiasis 09


y definiciones 01
3.l. Epidemiologia 09
3.2. Manifestaciones clínicas
1.l. Definiciones 01
1.2.
y su manejo agudo 10
Diagnóstico diferencial
3.3. Evaluación y tratamiento
de la hematuria macroscópica 02
de la litiasis renal 11

02. Infecciones
04. Tumores renales 15
del tracto urinario.
Cistitis intersticial 03 4.1. Carcinoma de células renales
(adenocarcinoma renal, hipernefroma) 15
2.l. Patogénesis y etiología 03 4.2. Otros tumores 17
2.2. Diagnóstico 04
2.3. Diferentes ITU y su tratamiento 04
2.4. Tuberculosis genitourinaria 07 05. Hiperplasia
2.5. Cistitis intersticial 07
y carcinoma prostático 19

S.l. Hiperplasia prostática benigna 19


5.2. Carcinoma prostático 21

VI
06. Carcinomas 09. Uropatía obstructiva 38
del tracto urinario 26
9.l. Características 38
9.2. Clínica 38
6.l. Carcinoma vesical 26
9.3. Diagnóstico 39
6.2. Tumores del tracto urinario superior 28
9.4. Tratamiento 39

07. Tumores testiculares 31


1 0. Disfunción eréctil 40
7.1. Etiología y epidemiología 31
10.1. Introducción 40
7.2. Anatomía patológica 32
10.2. Prevalencia 40
7.3. Clínica 32
10.3. Etiología 40
7.4. Diagnóstico 32
10.4. Factores de riesgo 40
7.5- Diagnóstico diferencial 33
10.5- Diagnóstico 41
7.6. Tratamiento 34
10.6. Tratamiento 41

08. Trasplante renal 36


Bibl iografía 42
8.l. Indicaciones 36
8.2. Contraindicaciones 36
8.3. Complicaciones 37

VII
01 •

SEMIOLOGíA UROLÓGICA
y DEFINICIONES

Orientación Aspectos esenciales

MIR
Este tema no se ha preguntado
en el MIR de forma directa.
GJ La causa más frecuente de hematuria microscópica es la litiasis (población general, ambos sexos).

Puede ayudar a obtener una la causa más común de hematuria microscópica en varones de más de 50 años es la hiperplasia benigna
visión general de la materia
III
de próstata.
ya asociar algunos hallazgos
a patologras concretas, pero la hematuria con coágulos indica un problema urológico.
no se debe emplear en él
demasiado tiempo. la causa más habitual de hematuria es la cistitis hemorrágica, pero 10 primero a descartar es el tumor uro­
leJia!.

IIJ Los hematíes dismórficos en el sedimento orientan a nefropalía de origen glomerular.

1 . 1 . Defin iciones

Hematuria microscópica: presencia de más de 5 hematíes por campo. La causa más frecuente en ambos sexos
es la litiasis. La causa mas común en varones mayores de 50 años es la hiperplasia benigna de próstata.
Hematuria macroscópica: orina de aspecto rojizo a simple vista debido a la presencia de más de 50 hematíes
por campo. En los pacientes fumadores, en ausencia de otros síntomas, se debe sospechar tumor urotelial.
Piuria: presencia de más de 10 leucocitos por campo. Altamente ¡nespecífica, pero en presencia de síntomas
urinarios, hay que sospechar infección.
Síndrome miccional: presencia de polaquiuria (aumento en la frecuencia miccional), urgencia miccional (ne­
cesidad imperiosa e irrefrenable de orinar) y disuria (molestias urinarias inespecíficas referidas como ardor,
escozor, etc.).
Incontinencia urinaria: pérdidas involuntarias de orina (Tabla 1 l. Existen cuatro tipos principales:

INCONTINENCIA INCONTINENCIA
SINTOMAS
DE URGENCIA DE ESFUERZO

Urgencia (deseo rwpentIno de ortnar) Si No

Aumento de laheuencla mlcdonal Si No

Copoc:ldod de __ .. bollo después de _ 01 deseo de orinar No SI

�_Ir"boIio_la_ sr Generalmente no

Escape _la lICtMdad fIsIca No Si

eanddod de orIno ....poda


. en coda opIlIOdlo de IncontI"'''cla Abundante. si se produce Generalmente escasa

Tabla 1. Diagnóstico diferencial de la cl ínica de incontinencia urinaria femenina

Continua: de día y de noche, en todas las posiciones. La causa más frecuente es la fístula urinaria (en pacientes
con antecedentes quirúrgicos previos) y la segunda, el uréter ectópico (que es la causa más frecuente en niñas).
De esfuerzo: se desencadena con el aumento de presión abdominal (al reír, toser, cargar con peso). Ge­
neralmente se produce por un déficit de soporte de la musculatura perineal (por ello es recomendable
? Preguntas
revisar los antecedentes obstétricos, pacientes obesas, pacientes añosas, ete.).

- No hay pregunta s MIR


De urgencia: el paciente siente ganas de orinar, pero no le da tiempo a l legar al baño (provocadas por
representativas. contracciones involuntarias del músculo detrusor).
Manual CTO de Medicina y Cirugfa, 8.a edición

Mixta: generalmente es una combinación de las dos anteriores.


1 .2. Diagnóstico d i ferencia l
Paradójica: escape de orina debido a la sobreclistensión vesical.
El ejemplo característico es el paciente prostático con retención d e l a hematuria macroscópica
urinaria. La presión i ntravesical supera la presión de cierre del
esfínter uretral, produciéndose un escape de orina paradójico
(no puede orinar y, sin embargo, se le escapa la orina). Según el momento de aparición:
Inicial: sangrado uretral o prostático.

• RECUERDA Final: sangrado del cuello vesical.


Total: vesical o del tracto urinario alto. Se debe recordar que un
los cilindros hemáticos aparecen en las glomerulonefritis que producen
síndrome nefrítico, como en la postinfecciosa. sangrado importante de cualquier parte del aparato genitourina­
rio puede provocar hematuria total.
Hematuria con coágulos: indica un problema urológico. La cau­
Enuresis: pérdidas de orina exclusivamente durante el sueño. Si el sa más frecuente en mujeres es la cistitis hemorrágica, aunque la
niño es mayor de 6 años, debe ser estudiado. primera causa a descartar es una neoplasia urotelial, máxime en el
Crisis renoureteral: dolor lumbar frecuentemente irradiado a geni­ paciente fumador.
tales, de carácter agudo, cuya intensidad no se modifica por los Hematuria por nefropatía médica: no suele tener coágulos, y puede
cambios pastura les, y que se suele acompañar de náuseas, vómitos ir acompañada de cierto grado de proteinuria, así como de cilindros
y malestar general. Es muy poco frecuente que sea bilateral. eritrocitarios o de hematíes dismórficos en el sedimento urinario.

2
02.
INFECCIONES DEL TRACTO URINARIO.
CISTITIS INTERSTICIAL

Orientación

MIR
la causa más frec uente de infección del tracto urinario OTU) es Escherichia coli, tanto a nivel comunitario
Este tema es el más importante
como nosocomial.
de esta asignatura. Se debe
conocer muy bien, pues dos El origen más frecuente de uretritis es Chlamydia lrachomalis.
o tres preguntas son habituales
en el examen. El estudio de las la causa habitual de orquiepididimitis depende de la edad: Chlamydia y gonococo si es menor de 35 años;
preguntas de otros años suele enterobacterias, si es mayor de esa edad.
ser de gran ayuda, ya que son
baStlnle repetitivas. No se la causa más frecuente de absceso renal en el UDVP es Staphylococcus aureus.
debe bajar la guardia con l a
tuberculosis genitaurinaria n i El diagnóstico definitivo de ITU es microbiológico: más de '05 UFClm1. No obstante, este criterio varía con
e n la cistitis intersticial. Hay el sistema de recogida.
que formar una imagen mental
típica para reconocerlas en Si se recoge la muestra urinaria mediante punción suprapúbica, cualquier número de bacterias es significativo.
caso clínico, que es como
la bacteriuria asintomática se trata en gestantes, menores de 5 años, inmunodeprimidos, previamente a la
suelen preguntarlas.
o cirugía urológica, o si la especie implicada es Proteus.

� Los sistemas de drenaje cerrados son preferibles a los abiertos, pues la lasa de infección es menor.

La infección del tracto urinario (lTU) puede clasificarse de varias formas. Se puede hacer una división anatómica
entre las ITU altas (infecciones renales) y las ITU bajas (cistouretritis, prostatitis). Asimismo, la clasificación pue­
de basarse en la asociación o no de complicaciones. Una ITU no complicada es un cuadro clínico caracterizado
por la presencia de escozor miccional, urgencia y frecuencia, acompañado o no por hematuria terminal, dolor
hipogástrico, y más raramente, febrícula. Dentro de este grupo se podrían incluir las pielonefritis no complica­
das, que se presentan como cuadros febriles con hipersensibilidad en fosa lumbar, fiebre, náuseas o vómitos, y
sin los factores que convierten la [TU en "complicada", como son: presencia de catéteres, uropatía obstructiva,
reflujo vesicoureteral, anomalías anatómicas, insuficiencia renal o trasplante renal. La ITU en el varón debe
considerarse esencialmente "complicada" de entrada.

La reaparición de una infección tras el tratamiento puede deberse a reinfección o recidiva. El primer término expre­
sa la infección nueva por un germen distinto al inicial, mientras que recidiva indica infección por el mismo germen.
Esta última es mucho más infrecuente que la reinfección y puede estar ocasionada por litiasis infectiva, prostatitis
crónica, fístulas vaginales o intestinales, divertículos vesicales infectados, cuerpos extraños, necrosis papilar infec­
tada y otras causas que generan un reservario de microorganismos que difícilmente se eliminan con el antibiótico.

2 . 1 . Patogénesis y etiolog ía

Existen tres posibles vías por las que los microorganismos pueden alcanzar el tracto urinario: hematógena,
III Preguntas linfática y ascendente. La vía l infática carece de i mportancia real. La diseminación hematógena tampoco es
ferecuente. La más común es la ascendente i niciada en la uretra. Probablemente por esta razón es mucho más
- MIR 08-09,98
habitual la ITU en mujeres, dado que su uretra es muy corta y ancha, y por ello favorece el paso de microorga­
- MIR 05-06, 230
-MIR 03-04, 78,8 4 nismos hacia niveles más altos del TGU.
- MIR 02-03, 1 34,1 74
-MIR OJ-02 , 103, 1 06
-MIR OO-01F,1 34, 1 45 Otro dato que apoya la importancia de la vía ascendente es la frecuencia de i nfección tras el cateterismo
- MIR 99-00, 1 35 uretral, que es del 1 % en los pacientes ambulantes, y en tres o cuatro días alcanza a casi la totalidad de los
- MIR 99-00f,1 1 9
- MIR 98-99F, 1 1 8
pacientes sondados con sistemas de drenaje abiertos. En los enfermos hospitalizados, el riesgo de i nfección
- MIR 97-98, 26,206, 2 1 5 alcanza un 5% por cada día de sondaje, incluso con sistemas cerrados (MIR 0 1 -02, 103).

3
Manual CTO de Medicina y Cirugía, 8.a edición

Una vez que las bacterias han alcanzado el tracto urinario, tres factores 2.3. Diferentes ITU y su trata m ie nto
determinan el desarrollo de la infección:
La virulencia del microorganismo.
El tamaño del inóculo.
Los mecanismos de defensa del huésped. En el tratamiento de la ITU lógicamente es fundamental el empleo de
antimicrobianos. El número de éstos empleado es elevado y las pautas
La mayoría de las infecciones en la comunidad están producidas por de tratamiento muy variables. A continuación, se repasarán las opcio­
gérmenes gramnegativos, principalmente E. coli (MIR 05-06, 230), res­ nes terapéuticas según el tipo de ITU a la que uno se enfrente.
ponsable del 85% y, en menor proporción, Proteus, Klebsie/la o Pseu­
domonas (MIR 97-98, 2 1 5). Entre los grampositivos, únicamente el Sta­
phylococcus saprophyticus tiene relevancia, produciendo el 10- 1 5% Bacteriuria asi ntomática
de las ITU en mujeres jóvenes (segundo germen más frecuente en esta
población).
Definida como bacteriuria significativa (105 UFC/ml) en a l menos dos
Alrededor del 30% de las mujeres con clínica miccional presentan re­ urocultivos con el mismo germen, tomados con una semana de dife­
cuentos menores de 105 colonias por mililitro ( 1 05 UFC/ml); de éstas, rencia en ausencia de síntomas. la bacteriuria asintomática no debe
tres cuartas partes presentan piuria; en el resto, existen pocos datos tratarse salvo en los casos en los que conlleva un riesgo de i nfección
que demuestren infección, y en general se tratan según la clínica. En clínica o daño orgánico, como ocurre en niños menores de 5 años,
la orina de las pacientes sintomáticas con piuria, se pueden encontrar tengan o no patología urológica asociada. Asimismo, debe ser tratada
(considerándose infección activa) recuentos más bajos (1 02- 1 04) de en el embarazo (MIR 08-09, 98 ; M I R 03-04, 78), en pacientes inmu­
los patógenos habituales. En otras ocasiones, el cuadro se justifica por nodeprimidos, como profilaxis previa a una cirugía urológica y en los
la presencia de uretritis causada por N. gonorrhoeae o C. trachomatis. casos de bacteriuria por Proteus (MIR 02-03, 1 34) (Tabla 2).
El papel patógeno de gérmenes como U. urealyticum o Mycoplasma
hominis está mal definido, ya que se desconoce su potencial como
Embarazadas
uropatógenos aislados (MIR 99-00F, 119).
Inmunodeprimidos
Previamente a cirugía urológica
En las i nfecciones nosocomiales, los gérmenes gramnegativos conti­
Bacteriuria por Proteus
núan siendo los más frecuentes. Si bien E. coli es el más habitual, su
frecuencia desciende hasta el 50% y adquieren mayor importancia Tabla 2. Bacterruria asintomática: indicaciones de tratamiento
Proteus, Klebsiella, Pseudomonas, Enlerobacler y Serratia (MIR 03-04,
84). El 25% restante está ocasionado por gérmenes grampositivos como
estreptococos y estafilococos. Candida albicans puede aparecer prin­
cipalmente en pacientes diabéticos, cateterizados o con tratamientos
D RECUERDA
Proteus es intrínsecamente resistente ti las nitrofurantoínas, ya que al­
antibióticos prolongados. caliniza la orina gracias a su ureasa, y este grupo de antibióticos única­
mente es útil en medio ácido.

la afectación del tracto urinario superior parece también producirse


por ascenso de los gérmenes a lo largo del uréter. La diferenciación,
aunque poco específica, se debe basar en los hallazgos clínicos (fiebre, En el caso de los pacientes sondados permanentemente, la presencia
dolor lumbar, escalofríos) y analítica elemental ( leucocitosis, velocidad de bacteriuria asintomática no es una indicación de tratamiento y,
de sedimentación alta). actualmente, incluso es dudosa la recomendación clásica de empleo
profiláctico de algún antibiótico, previo a la sustitución del catéter, a
fin de contrarrestar la posible diseminación hematógena del germen
producida por la manipulación (las ú ltimas guías clínicas ya no lo reco­
2.2. D i a g nóstico miendan). Sí es, sin embargo, i ndicación de tratamiento la bacteriuria
persistente a los 3-5 días de haber retirado una sonda vesical. En aque­
llos pacientes en los que la sonda no pueda ser retirada, el tratamien�
E l d i agnóstico de ITU, además de la clínica, se define por el cultivo to de las bacteriurias asi ntomáticas no suele ser efectivo, y puede dar
de orina. Dado que es frecuente el crecimiento de bacterias que han lugar a selección de cepas resistentes. En estos pacientes sólo se debe
contaminado las muestras, se utiliza u n criterio estadístico sobre la iniciar tratamiento si presentan alto riesgo de desarrollar bacteriemia o
base del recuento de colonias del urocultivo, considerando como si la bacteriuria se hace sintomática.
significativo clásicamente el crecimi ento de más de 1 05 colonias por
m i l i l itro (MIR 97-98, 206). En determinadas circunstancias, recuen­ En el resto de los casos, únicamente con la concurrencia de factores
tos de colonias menores pueden ser suficientes: recuentos de 1 03 particulares, se debe tratar la bacteriuria, y siempre sobre la base del
UFC/ml en mujeres sintomáticas, más de 1 04 en pielonefritis c l ínicas estudio de sensibil idades (MIR 98-99F, 1 1 8).
o en varones, y más de 1 02 en muestras de cateterismos l i mpios
o cualquier recuento, si se recoge mediante punción-aspiración su­
prapúbica (MIR 97-98, 26). Cifras mayores de 105 U FC/ml pueden ITU baja en mujeres
igualmente reflejar contaminación, principalmente si crecen dos o
más especies.
Puede realizarse un tratamiento convencional de siete días o bien u n
En el adulto, la presencia de piuria (más de 1 0 leucocitos/mm3) se rela­ curso corto e n monodosis o en régimen d e tres días. L a ventaja d e éstos
ciona estrechamente con la ITU en presencia de síntomas, no así en el es el menor coste económico y la menor incidencia de efectos adver­
niño, en el que puede acompañar a los cuadros febriles. sos. Su desventaja es la mayor incidencia de recurrencias tempranas, a l

4
urolog¡a a
no afectar apenas a los reservorios vaginal e intestinal de uropatógenos. El antibiótico empleado se seleccionará, por supuesto, sobre l a
Aun con todo, por las ventajas mencionadas, la pauta preferida actual­ base del cultivo y d e l antibiograma, y cuando s e i n icie d e forma
mente es el tratamiento de tres días. empírica, habrá que tener en cuenta factores que orienten hacia
el germen causante: mayor incidencia de Pseudomonas en perso­
Los antibióticos de elección son el cotrimoxazol, las fluoroquinolonas nas diabéticas y enfermos de UVI, estafilococo en adictos a drogas
y la amoxicilina-ácido clavulánico, fosfomicina o n itrofurantoína. parenterales (MIR 03-04, 84), Proteus en pacientes con litiasis in­
fectiva, presencia de sondas, catéteres, tratamientos antibióticos
En mujeres embarazadas se recomiendan las pautas largas de trata­ previos, etc.
miento (siete días), evitando el uso de sulfamidas al final del embarazo
por el riesgo incrementado de kernicterus, y el empleo de quinolonas En la evaluación del paciente con pielonefritis y mala respuesta a tra­
por el daño producido sobre el cartílago de crecimiento fetal. Tampoco tamiento inicial, es recomendable la realización de una ecografía para
se emplearán pautas cortas en caso de sospecha de pielonefritis, pre­ descartar obstrucción o litiasis (M I R 00-01 F, 134).
sencia de cálculos o anomalías de la vía urinaria, o bien infecciones
previas por microorganismos resistentes a los antibióticos. ITU en varones

ITU recurrente Cualquier ITU en varón debe considerarse como complicada inicial­
mente ya que hay que asumir que existe afectación del tejido prostá­
tico, renal o que existen problemas concomitantes como obstrucción
Aparición de cuatro o más episodios al año. Se puede realizar profi­ urinaria, litiasis o malformaciones urológicas. Por todo ello, el trata­
laxis con cotrimoxazol O una fluoroquinolona (en función de la sensi­ miento debe ser más prolongado (mínimo una semana), no siendo ade­
bilidad del germen aislado en el último episodio) en dosis única, días cuados los cursos cortos de tratamiento.
alternos, durante seis meses. Si tras la retirada se presentaran nuevas
recurrencias, puede reinstaurarse el tratamiento durante periodos más
prolongados (1-2 años). Es aconsejable la ingesta abundante de agua Prostatitis
y realizar micciones frecuentes y cumplir una serie de reglas básicas
higiénico-dietéticas.
La infección aguda del tej ido prostático se presenta como un cuadro
Si los episodios tienen relación con el coito, se puede administrar un séptico con afectación general del paciente, fiebre elevada, síndrome
comprimido de cotrimoxazol o una quinolona después del mismo. En miccional, artromialgias y dificultad miccional (Tabla 3). En el examen
mujeres posmenopáusicas, el tratamiento con estrógenos tópicos vagi­ rectal, la próstata aparece muy dolorosa e inflamada. El germen más
nales disminuye la frecuencia de i nfecciones. habitual es E. coli. Durante la inflamación aguda, los antibióticos pe­
netran adecuadamente, pero una vez que ésta cede, la penetración es

• RECUERDA más pobre. Por ello, se deben utilizar cursos largos de tratamiento (3-4
semanas) para i n tentar evitar la persistencia de focos que den pie a una
Staphylococcus saprophylicus se ha relacionado con ITU en mujeres
jóvenes sexualmente activas. prostatitis crónica. Entre los antimicrobianos empleados, las fluoroqui­
nolonas son las que mejor difunden al tejido prostático .

Pielone!ritis aguda no complicada


• RECUERDA
En pacientes con SIDA, Cryptococcus neoformans puede ser una causa
de prostatitis, ya que se elimina a través de la orina.
En los casos de gravedad leve-moderada, puede plantearse terapéu­
tica oral con cotrimoxazol (en desuso en nuestro medio por el ele­
vado índice de resistencias), fluoroquinolonas o B- Iactámicos. En La prostatitis crónica bacteriana suele presentarse como molestias
pacientes graves u hospita l i zados es preciso tratamiento parenteral, perineales o genitales, síntomas i rritativos (polaquiuria, tenesmo, es­
y el espectro de antimicrobianos incluye ampicilina (enterococo), cozor) y episodios de ITU recurrentes causados por el mismo organis­
ureidopen icilinas (Pseudomonas), cefalosporinas de segunda o ter­ mo. En el líquido prostático se evidencian más de 10 leucocitos por
cera generación, e incluso aminoglucósidos. Nunca se emplearán campo de gran aumento, y macrófagos que contienen cuerpos ovales
pautas cortas. grasos.

CULTIVO LIQUIDO
ETIOLOGIA ClINICA H 'ITU CULTIVO ORINA LIQUIDO PROSTATlCO TRATAMIENTO
PROSTATICO

Cotrimoxazol,
-
E.coH Cuadro séptico + + Nunca hacer masaje prostático ni sondaje fluoroquinolonas
ogudo
4 semanas
Prostadtls Irritativo con Cotrimoxazol,
crónica E.coJ/ reagudizaciones, sin + +/- > 10 leucocitos/campo + fluoroqulnolonas
- fiebre ni leucocitosis 6-12 semanas
-
Ureaplasma Cronicidad,
crónica - - > 10 leucocitos/campo - Doxicidina
Myeoplasma empeoramiento
no*-no
a-bloqueantes
- Desconocida Oscilante - -
< 10 leucocitos/campo -
Relajantes musculares

Tabla 3. Oiagnóstko diferencial de las prostatitis

5
Manual CTO de Medicina y Cirugía, B." edición

El tratamiento debe estar guiado por los cultivos, tanto de orina como
de fluido obtenido por masaje prostático, y prolongarse entre 4 y 1 6
semanas. Cuando se encuentra a un paciente con datos de prostatitis
crónica y signos inflamatorios en el líquido prostático, pero sin historia
documentada de ITU y con cultivos negativos, el cuadro se denomina
prostatitis no bacteriana. En ocasiones, el responsable puede ser U.
urealylicum o M . hominis, pudiendo ser tratados estos casos con doxi­
ciclina o eritromicina, sobre esta sospecha.

Se denomina prostatodinia a un cuadro clínico similar donde predo­


minan las molestias perineales o genitales con cultivos negativos y
menos de 1 0 leucocitos por campo en el líquido prostático. Su cau­
sa es desconocida y el tratamiento difícil, empleándose actualmente
p-bloqueantes o relajantes musculares como terapia inicial.

Orqu iepididimitis

En varones adultos menores de 3 5 años e s considerada, en el plano


teórico, una enfermedad d e transmisión sexual, siendo los agentes
más frecuentes Chlam ydia trachomatis y Neisseria gonorrhoeae (Ta­
bla 4). Por encima de 35 años, los microorgan i smos más frecuentes
son las enterobacterias. El tratamiento puede l l evarse a cabo con las
sigui entes pautas: 1 ) quinolonas, 2) ceftriaxona en dosis única i . m .
( 1 2 5-250 mg) más 1 0 días de doxiciclina ( 1 00 mgl1 2 h/7 días), se
aplicará esta pauta en aquellos casos en los que se sospeche ETS
(MIR 99-00, 1 35).

Absceso renal

Los abscesos medulares o corticales suelen proceder de un foco de pie­


lonefritis contiguo O de diseminación hematógena de s. aureus/ proce­
dente de focos cutáneos en sujetos adictos a drogas por vía parenteral .
E l urocultivo e n este último caso puede ser negativo. El diagnóstico
más fiable se realiza mediante TC Deben tratarse con antibióticos El diagnóstico es similar al absceso renal, y su tratamiento pasa por
por vía intravenosa y, dependiendo del tamaño y de la evolución, el drenaje percutáneo o quirúrgico, con la adecuada cobertura anti­
se hace obligatorio el drenaje mediante punción percutánea O quirúr­ biótica.
gicamente.

ITU asociada a catéteres


Absceso pe,i"enal

Se localiza entre la cápsula renal y la fascia de Gerota. Lo más fre­ La ITU es la infección hospitalaria más frecuente, y los catéteres urina­
cuente es que un absceso cortical se abra a este espacio, pero pue­ rios la principal fuente de sepsis. Se calcula que el 1 % de cateterismos
d e ocurrir también por diseminación hematógena. El germen más ambulatorios transitorios sufren una ITU posterior y que la mayoría de
frecuente es E. coli, y enfermos con catéter permanente presentan una bacteriuria significati­
D RECUERDA S. aureus en los casos va al cuarto día de su colocación. Esta bacteriuria puede hacerse sinto­
S. oureus es también la causa más frecuente
de diseminación he­ mática en forma de cuadros de cistitis, hematuria o episodios febriles,
de endocarditis Infecciosa.
matógena (Figura 1). muchas veces autolimitados.

ENFERMEDAD ETIOLOGIA LESiÓN TIPICA DIAGNÓSTICO TRATAMIENTO

Neisserio gonorrhoeoe Asintomática (�) Contacto < 5 dlas Ceftriaxona o


Exudación uretral matutina (1;) Gram de exudado cervical espectinomicina (no en
Epididimoprostatitis, salpingitis, síndrome Cultivo en medio de Thayer-Martin faríngeas)Ciprofloxacino
Uretritis gonocócIco
Fitz-Hugh-Curtis, gonococemia diseminada
(déficit C5-C.. menstruación, embarazo,
auxotipo AHU)

Chlomydio trochomotis, Similar a las UG, pero con menos signos Contacto 7-15 dias. Excluir gonorrea Tetraciclinas o macrólidos
lInIIrItIs no __ Ureoplosmo ureolyticum y síntomas por Gram y cultivo. C. inclusión-
Epldidimitis, proctitis, cervicitis, EIP Giemsa IFD, medios celulares

Tabla 4. Diagnóstico diferencial de las uretritis

6
urologia a
Entre los factores que aumentan el riesgo de ITU asociada a catéter
Clínica
urinario se pueden enumerar: 1) sexo femenino, 2) edad avanzada, 3)
mala técnica de sondaje, 41 sistemas de drenaje abiertos y 51 falta de
higiene local. Los hallazgos clínicos son escasos. En el 70% de los pacientes, los sín­
tomas son leves. Lo más frecuente es la aparición de microhematuria,
Entre los antibióticos disponibles, parece que las quinolonas son los dolor vago en flanco o cólico renal. La afectación vesical, sin embargo,
que mejor eliminan la película biológica de los catéteres infectados, sí produce sintomatología florida con un síndrome cistítico rebelde,
favoreciendo así el tratamiento de la infección; en cualquier caso, éste donde la polaquiuria (secundaria a la disminución de la capacidad
únicamente se recomienda si existe sintomatología o en el momento de vesical) es lo más llamativo. En varones, es frecuente la aparición de
la retirada del catéter, por el mayor riesgo de ITU sintomática y sepsis. una orquiepididimitis crónica que no responde a la terapia habitual.

En el 90% de los pacientes, el análisis urinario es anormal. Típicamente


aparece piuria ácida con urocultivo negativo. La prueba de laborato­
2.4. Tu berc u l os is g e n itou r i n a ri a rio más importante es el cultivo de M. tuberculosis en medio selectivo
(Lowenstein), ya que los medios de tinción rápida (Ziehl, auramina),
aunque válidos, pueden dar falsos positivos por contaminación con M.
Generalmente está ocasionada por Mycobacterium tuberculosis. El apa­ smegmatis (MtR 02-03, 174).
rato genitourinario es el sitio más frecuente de afectación extrapulmonar
(tras la adenitis tuberculosa). Un 5% de los pacientes con tuberculosis
activa presentan afectación del tracto genitourinario (Figura 2). Diag nóstico

Tuberculosis
miliar El cultivo en medio de Lowenstein es positivo en el 90% de los pa­
cientes con enfermedad activa, aunque deben obtenerse, al menos, tres
muestras de días diferentes para mejorar la sensibi lidad, ya que el paso
de bacilos a orina no es constante. Actualmente, lo más rentable es rea­
lizar una PCR de orina en busca del ARN del bacilo.

Radiológicamente, el 90% de los pacientes presentan urogramas altera­


Amputación Pionefrosis dos. El hallazgo más sugestivo es la presencia de cavidades que comu­
catieial nican con el sistema colector. Inicialmente estas cavidades son mínimas
y dan un aspecto "mordisqueado" a los cálices. Según la enfermedad
avanza, pueden encontrarse estenosis infundibulares, ureteropiélicas, en
Trompa
Mierovejiga
unión ureterovesical o vejigas pequeñas de aspecto rígido. En el punto
más evolucionado de la enfermedad, el riñón puede encontrarse anula­
do, disminuido de tamaño y con calcificaciones parenquimatosas.

Próstata
y vesfeulas Trata miento
seminales

Estenosis El tratamiento médico de la enfermedad activa no difiere sustancial­


ureteral distal
Epididimitis mente del de la tuberculosis pulmonar en cuanto a fármacos y periodo
de tratamiento. Puede ser necesario el tratamiento quirúrgico, depen­
diendo de la complicación asociada, generalmente estenosis de la vía
Figura 2. Lesiones de la tuberculosis genitourlnaria excretora e h idronefrosis. En caso de riñón no funcionante por lesión
extensa del parénquima, puede ser precisa la nefrectomía.

Tras la inhalación del bacilo, se produce una diseminación hematógena


(primoinfección) con siembra de bacilos en ambos riñones en el 90% de 11 :�����?n�, isoniacida y pirazinamida son los tres antibióticos más em­
los casos. Sin embargo, la enfermedad clínica generalmente es uni lateral. pleados en la tuberculosis.

El periodo de latencia entre la "siembra" y la enfermedad clínica oscila


entre 1 O Y 40 años, afectando principalmente a pacientes por debajo de
los 50 años. La lesión inicial microscópica se localiza en los gloméru­
los en forma de granulomas microscópicos. Al avanzar la enfermedad, 2.5. C istitis intersticial
se produce afectación más distal hasta la aparición de una papilitis
necrotizante, momento en el cual ya puede existir paso de bacilos a la
vía excretora donde, por procesos inflamatorios, ocasionará estenosis Aunque no es u n cuadro infeccioso, s e incluye e n el presente capítulo
a nivel de los infundíbulos caliciales, pelvis y uréter, con hidronefrosis esta entidad inflamatoria vesical de origen desconocido. En este senti­
secundaria. Las lesiones renales pueden cavitarse y calcificarse, y lle­ do, se esgrimen dos teorías no demostradas: por un lado, la teoría au­
gar a producir una destrucción total del parénquima (fenómeno que se toinmunitaria, y por otro, la de un déficit en el recubrimiento urotelial
denomina "riñón mastic" ). por glucosaminoglucanos.

7
Manual elO de Medici na y Cirugía, s.a edición

Cl ínica litiasis o tumor vesical) apoyado en los hallazgos cistoscópicos suges­


tivos: 1) petequias submucosas, principalmente trigonales, que apare­
Suele presentarse en mujeres entre 30 y 70 ar1os, como un cuadro cis­ cen al distender la vejiga (glomerulaciones), 2) úlceras de Hunner. La
títico crónico en el que destacan disuria, polaquiuria con nicturia y biopsia vesical, además de descartar la presencia de carcinoma in situ
dolor suprapúbico, acompañados en ocasiones de hematuria (20-30%) u otra patología, revela en algunos casos, un infiltrado intersticial de
(MIR 01-02, 1 06) . mastocitos (MIR 00-01 F, 1 45).

D RECUERDA Trata miento


Existen muchas más causas de síndrome cistítico: cistitis aguda, tuberculo­
sis, carcinoma in situ, etc.

Aunque esta enfermedad raramente supone una amenaza para la vida de


la paciente, su morbilidad es elevada. Desgraciadamente, las diversas al­
Diag nóstico ternativas de tratamiento únicamente pueden encaminarse a una mejoría
sintomática, en la mayoría de los casos con resultados discretos; 1 ) dis­
tensión hidráulica vesical, 2) amitriptilina oral, 3) instilación con dime­
El diagnóstico es básicamente por exclusión de otra patología que tilsulfóxido [DMSOJ, 4) corticoides tópicos o sistémicos, 5) denervación
pueda ocasionM un cuadro similar (infección bacteriana, tuberculosis, vesical, 6) cistoplastias �e aumento, y en último término, 7) cistectomía.

Casos clínicos representativos

Un prostático, sin otros problemas de salud, portador de sonda uretral pennanente, 4) TBe urogenital.
presenta bacleriuria (> 10' unidades formadoras de colonias) en dos uroculti\los. 5) Pielonefritis crónica por P. aeruginosa.
¿Cuál es la actitud terapéutica más con\lenientel
RC: 4
1 ) Tratamiento antibiótico de amplio espectro.
2) Tratamiento antibiótico según antibiograma.
3) Cambio de sonda urinaria exclusivamente. Una paciente de 27 años acude al servido de Urgencias por dolor en fosa renal
4) Antisépticos en vejiga urinaria. derecha, fiebre de 39 "e, escalofríos y síndrome micdonal acompañante. Es alérgica
5) Cambio de sond..l urinaria y tratamiento antibiótico. a penicilinas. Señale la respuesta correcta:

RC: 3 1) No será necesario descartar patología urinaria obstrUd;va en este caso, ya que
presenta un claro síndrome miccional.
2) Para poder hacer el diagnóstico de pielonefritis se deberá conocer primero 10$
Ante un paciente de 24 años, que presenta fiebre alta con dolor, inflamación y datos referidos a la función renal.
enrojecimiento testicular izquierdo, ¿cuál de las siguientes afirmaciones es INCO­ 3) Se deberá iniciar tratamiento empírico con un �-Iactámico.
RRECTA? 4) Si en las primeras horas evoluciona favorablemente podrá continuar el tratamien­
to de forma ambulante.
1) El diagnóstico más probable es el de epididimitis. 5) El mejor tratamiento disponible es la administración intramuscular de aminoglu­
2) los patógenos más frecuentcs son Chlamydia lrachomalis y Neisseria gonorrhoeae. cósidos.
3) El tratamiento de elección es vancomicína + gentamicina.
4) El tratamiento de elección puede ser ofloxacino. RC: 4
5) Un tratamiento alternativo es ceftriaxona en monooosis más 1 o días de doxiciclina.

MIR 99-00, 135; RC: 3 Un paciente de 83 años sondado de forma permanente acude a la consulta tras
detectársele dos cultivos positivos lomados con una semana de diferencia. Asegura
encontrarse asintomático. la actítud más adecuada será:
Ante un paciente que presenta febrícula persistente, crisis renoureterales bre\les,
piuria est�ril, orina con pH ácido, microhematuria persistente, con citología uri­ 1) Iniciar tratamiento antibiótico según antibiograma de los cuhivO$ obtenidos.
naria negati\la y epidídimos indurados .f.en qué enfermedad se debe pensar pri­ 2) Tranquilizar al paciente y seguir con su pauta habitual de recambio de sonda.
mero? 3) Realizar cambio de sonda de forma inmediata con tratamiento antibiótico.
4) Realizar cambio de sonda de forma inmediata con profila)(is antibiótica de 4 días.
1) Sarcoidasis. 5) Retirar la sonda y colocar cistotomía suprapúbica.
2) Carcinoma vesical.
3) Carcinoma renal. RC: 2

8
03.
UROLlTIASIS

OrientaCIón

MIR
Tema fundamental en
esta asignatura. Se debe OJ Los cálculos más frecuentes son los de oxalato cálcico.
conocer muy bien la actitud
ante la litiasis en general,
ante los distintos tipos de
III Globalmente, la litiasis es más común en el varón, salvo las de estruvita, más comunes en mujeres.
cálculos y, especialmente,
todo lo relacionado con m La radiografía de abdomen no permite ver algunos cálculos, como los de urato. Sin embargo, la ecografía
puede verlos, independientemente de su composición.
el tratamiento. Es un tema
rentable y agradecido. así que Litiasis radiotransparentes: Sulfamidas, Indinavir, Urato, Xantinas (SIUX). las de cistina son radiolúcldas; y
hay que emplear el tiempo
el resto, radioopacas.
necesario. la tabla-resumen
de urolitiasis puede ser de
las tiazidas son útiles para la hipercalciuria idiopática.
gran ayuda.

los cálculos asociados a las resecciones ileales O a la enfermedad inflamatoria intestinal son de oxalato
cálcico.
Precipitan en medio ácido: ácido úrico y cistina. Precipitan en medio alcalino las que contienen fosfatos
(fosfato amónico magnésico o estruvita, y el fosfato cálcico).

En el tratamiento de la litiasis por ácido úrico es beneficioso alcalinizar la orina.

los cálculos de oxalato NO se ven alterados por el pH (al Oxal, el pH de la igual).

los cálculos de estruvita se relacionan con microorganismos productores de ureasa, como Proteus.

las contraindicaciones absolutas para la LEOC son: embarazo, infección activa y obstrucción de las vías
urinarias distal al cálculo.

3 . 1 . E p id e m iología
Son numerosas las sustancias que se han identificado formando parte de 105 cálculos. Su i ncidencia varía según
el país, e incluso según las áreas geográficas dentro del mismo país.

Se pueden distinguir seis grupos de componentes:


Oxalato cálcico.
Fosfato cálcico.
Fosfato no cálcico.
Compuestos purínicos (ácido úrico, urato amónico, urato sódico, xantina, 2 , 8 dihidroxiadenina).
Aminoácidos (cistina).
Otros (carbonato cálcico, sulfamidas, etc.).

[?J Preguntas
Los cálculos de oxalato cálcico son los más frecuentes, con cifras en torno al 65%, seguidos por los infectivos
-MIR 08-09, 93 y ácido úrico (alrededor del 15% cada uno). Fosfato cálcico un 5%, y los de cistina con una incidencia baja
- MIR 06-07, 93, 106
(1-3%).
-MIR 05-06, 104
-MIR 04-05, 104
- MIR 03-04, 44, 80 La tercera década es la edad media de aparición, por primera vez, de la litiasis salvo en los de cistina, que suelen
- MIR 02-03, 1 76
- MIR OO-01 , 1 1 7
ser de aparición más prematura.
- MIR 99-00, 1 89
- MIR 99-00F, 143, 146
- MIR 98-99, 135, 138
En España, la incidencia de litiasis alcanza al 4,2% de la población, con mayor afectación de varones que mu­
- MIR 98-99F, 144 jeres. Ú nicamente los cálculos infectivos tienen mayor incidencia en la mujer.

9
03.
UROLlTIASIS

OrientaCiÓn Aspectos esenciales

MIR
TerT'kl fundamental en
esta asignatura. Se debe los cálculos más frecuentes son los de oxalato cálcico.
conocer muy bien la actitud
ante la litiasis en general. Globalmente, la litiasis es más común en el varón, salvo las de estruvita, más comunes en mujeres.
ante los distintos tijX)S de
cálculos y. especialmente, la radiografía de abdomen no permite ver algunos cálculos, como los de urato. Sin embargo, la ecografía
todo lo relacionado con
puede verlos, independientemente de su composición.
el tratamiento. Es un tema
rentable y agradecido, así que Litiasis radiotransparentes: Sulfamidas, Indinavir, Urato, Xantinas (SIUX). las de cistina son radiolúcidas; y
hay que emplear el tiempo
el resto, radioopacas.
necesario. La tabla-resumen
de urolitiasis puede ser de
las tiazidas son útiles para la hipercalciuria idiopática.
gran ayuda.

los cálculos asociados a las resecciones ileales o a la enfermedad inflamatoria intestinal son de oxalato
cálcico.

o Precipitan en medio ácido: ácido úrico y cistina. Precipitan en medio alcalino las que contienen fosfatos
(fosfato amónico magnésico O estruvita, y el fosfato cálcico).

En el tratamiento de la litiasis por ácido úrico es beneficioso alcalinizar la orina.

los cálculos de oxalato NO se ven alterados por el pH (al Oxal, el pH de la igual).

los cálculos de estruvita se relacionan con microorganismos productores de ureasa, como Proteus.

Las contraindicaciones absolutas para la LEOC son: embarazo, infección activa y obstrucción de las vías
urinarias distal al cálculo.

3 . 1 . E p i d e m iología
Son numerosas las sustancias que se han identificado formando parte de los cálculos. Su incidencia varía según
el país, e incluso según las áreas geográficas dentro del mismo país.

Se pueden distinguir seis grupos de componentes:


Oxalato cálcico.
Fosfato cálcico.
Fosfato no cálcico.
Compuestos purínicos (ácido úrico, urato amónico, urato sódico, xantina, 2,8 dihidroxiadenina).
Aminoácidos (cistina).
Otros (carbonato cálcico, sulfamidas, ete.).
�U Preguntas
Los cálculos de oxalato cálcico son los más frecuentes, con cifras en torno al 65%, seguidos por los i nfectivos
- MIR 08-09, 9 3 y ácido úrico (alrededor del 1 5% cada uno). Fosfato cálcico un 5%, y los de cistina con una i ncidencia baja
- MIR 06-07, lJ3. 106
( 1 -3%).
- MIR 05-06, 104
- MIR 04-05, 104
- MIR 03-04, 44, 80 La tercera década es la edad media de aparición, por primera vez, de la litiasis salvo en los de cistina, que suelen
- MIR 02-OJ, ' 76
- MIR OO-Ol , 1 1 7 ser de aparición más prematura.
- MIR 99-00, 189
- MIR 99-00F, 143, 1 4 6
En España, la incidencia de l itiasis alcanza al 4,2% de la población, con mayor afectación de varones que mu­
- MIR 98-99, 1 3 5 , 1 38
- MIR 98-99F, 144 jeres. Ú nicamente los cálculos infectivos tienen mayor incidencia en la mujer.

9
Manual eTO de Medicina y Cirugía, B.a edición

o RECUERDA coraliforme o "en asta de venado" [Figura 4]), manifestándose no como


cólico, sino como infecciones urinarias de repetición, dolor lumbar
las infecciones urinarias son más frecuentes en mujeres que en varones.
Por eso los cálculos de estruvita también lo son. sordo, hematuria o incluso insuficiencia renal terminal.

La enfermedad litiásica recidiva en el 40% de los casos, con una media


de un nuevo cálculo cada dos o tres años. Por recidiva se entiende la apa­
rición de una nueva litiasis de la misma composición y en la misma lo­
calización, en un intervalo menor de cuatro años entre un cálculo y otro.

3.2. M a n ifestaciones c l ín i cas


y su manejo a g u d o

El dolor agudo del cólico renal es l a manifestación más típica de la li­


tiasis renal. El dolor se produce por la sobredistensión de la vía urinaria
tras la obstrucción de ésta por el cálculo. Es lógico, por tanto, que el
cálculo deba desplazarse desde su origen calicial para producir sinto­
matología aguda. Ocasionalmente se observan cuadros de dolor vago
renal en relación con litiasis caliciales no desplazadas.
Diagnóstico
El cólico renal o crisis renoureteral suele aparecer de forma progresi­
va sobre la fosa lumbar afectada, irradiándose por el flanco hacia la
ingle y los genitales (Figura 3). El paciente generalmente se encuentra El análisis básico de orina muestra generalmente hematuria y leuco­
afectado, con dolor que no cede con reposo, por lo que cambia de cituria. Una piuria importante apoyaría la posibil idad de infección
postura continuamente. Puede acompañarse de un cortejo vegetativo sobreañadida, aunque ninguno de estos datos es realmente determi­
con náuseas, vómitos y sudoración. El dolor irradiado hacia la ingle nante.
generalmente indica que el cálculo ha alcanzado el uréter. Cuando se
encuentra en vecindad de la vejiga, o bien dentro de ésta, puede apare­ Los cristales de oxalato cálcico d i h idralado aparecen como bipirá­
cer un cuadro irritativo, similar al síndrome miccional con polaquiuria, mides tetragonales a l observarlos con lupa b i nocular. Los de oxa­
disuria y tenesmo vesical. lato cálcico monohidratado aparecen como cristales alargados que
adoptan forma de empalizada, formando cálculos de estructura ra­
diada, con aspecto compacto y macizo. Entre los fosfatos cálcicos,
la brushita es el compuesto más ácido, formando cristales grandes
Sudoración en forma de abanico de color azul con luz polarizada. Las apatitas
tienen aspecto microgranular o esferocítico. El ácido úrico aparece
bajo la lupa como una desordenada aglomeración de cristales. En
algunos cálculos, los cristales están tan juntos que se asemejan a u n a
masa continua.

La estruvita (o fosfato amónico magnésico) es el componente más ca­


RX simple de abdomen
racterístico de los cálculos producidos por infección por gérmenes
con cálculo a nivel de l3
urealíticos. Sus cristales tienen formas prismáticas polimorfas, y rara­
mente se observan los cristales "en ataúd" que pueden hal larse en el
sedimento. La cistina se reconoce fácilmente por su aspecto acarame­
lado, formando cristales hexagonales en prismas o láminas.
Dolor de inicio
En teoría, el 90% de los cálculos son visibles en una radiografía simple
de abdomen, aunque este porcentaje es considerablemente menor en

Dolor irradiado Síndrome miccional


las radiografías urgentes sin preparación intestinal. Radiológicamente,
Hematuria la mayoría de los cálculos son radioopacos, exceptuando los de ácido
úrico y algunas otras composiciones infrecuentes (sulfamidas, xantina,
indinavir) (MIR 04-05, 1 04).

El estudio de imagen se completará mediante otras técnicas diag­


nósticas. La ecografía permitirá visualizar incluso las l itiasis radio­
Figura 3. Diagnóstico de urolitiasis
transparentes, con el inconveniente de no ser vistas aquellas ubi­
cadas en el trayecto ureteral (salvo las zonas cercanas a la vejiga o
Los cálculos infectivos de estruvita, y en menor medida, los de ácido al riñón). También se podrá evaluar el grado de hidronefrosis ( M I R
úrico y cistina pueden crecer modelando las cavidades renales ( l itiasis 99-00, 1 89).

10
urolOgia �
La urograffa ofrece información morfológica y funcional de ambos
riñones (Figura 5). Debe tenerse en cuenta que, durante el cólico
renal, puede observarse una anulación funcional, sin que signifi­
que necesariamente deterioro de dicha unidad renal. Mediante
esta técnica se puede diagnosticar todo tipo de cálculos, ya sean
radiotransparentes o radioopacos. El principal inconveniente de
este procedimiento es la introducción de contraste yodado, que está
contra indicado en los pacientes con alergia, creatin i na mayor de 2,
mieloma múltiple o deshidratación i mportante.

Según las guías clínicas, la urografía intravenosa (UIV) actualmente ha


sido desplazada por la Te helicoidal sin contraste, que se ha convertido
en el nuevo estudio de referencia para las litiasis. Aunque su alto coste
hace que todavía no esté extendido su uso, permite evaluar todo tipo
de cálculos.
Figura 6. Doble J derecho. litiasis ureteral derecha. litiasis coraliforme izquierda

Tratamiento

3.3. Eva l u ación y trata m iento


El manejo agudo del cólico renal se basa en el control del dolor.
Para esto, es preciso conseguir una disminución de la presión den­ de l a l itiasis ren a l
tro de la vía urinaria, Jo que puede hacerse, sobre todo, con anti­
inflamatorios, que disminuyen el dolor y la d iuresis a l i n hibir la
síntesis de prostaglandinas. Asimismo, se pueden usar espasmolíti­ Este apartado se puede dividir en dos partes. Por u n lado, el estudio
cos, que disminuyen la presión intraureteral al relajar la pared del de la litiasis con la finalidad de instaurar un tratamiento preventivo
uréter. de su formación, y por otro, el estudio y tratamiento de la litiasis ya
formada.
Existen una serie de situaciones en las que el cólico renal se convierte
en una urgencia que precisa de hospitalización, y eventualmente, de
manipulación invasiva: Estudio
Obstrucción grave, principalmente si se acompaña de litiasis mayor
y tratamiento preventivo
de 1 0 mm.
Fiebre elevada (mayor de 38 OC).
Dolor incontrolable.
Riñón único. La evaluación del paciente con litiasis se basa en un estudio meta­
bólico para determinar qué factores son modificables, en un intento
Asimismo, en pacientes diabéticos, por el mayor riesgo de com­ de evitar la recidiva (Tabla 5).
pl icaciones, es aconsejable, si no el i ngreso, a l menos una obser­

a RECUERDA
vación estricta. Una situación s i m i l a r ocurre durante el emba razo, Este estudio debe reservarse para aque­
donde una dilatación leve de la vía urinaria puede considerarse l l os pacientes con alta proba b i l idad de
la furosemida, al re­
" fisiológica", pero obstrucciones más importantes o la aparición recidiva, aunque cada vez más autores vés que las tiazidas,
de fiebre hacen aconsejable l a colocación de u n catéter u reteral indican que debe realizarse a todos Jos aumenta el calcio uri­
nario.
(Figura 6). pacientes (Tabla 6).

11
Manual CTO de Medicina y Cirugía, B.a edición

LITIASIS plante hepático, que suele ir unido al renal, aunque algunos casos
LITIASIS CALCICA LITIASIS LITIASIS
INFECCIOSA responden a piridoxina.
(OXALATO O FOSFATO) URICA CtSTINICA
(ESTRUVITA)

11 RECUERDA
· Hipercalciuria idiopática · Gota primaria . Cistinuria . Infecciones
· Hipercalciuria secundaria · Hemopatías por gérmenes
a hipercalcemia · Enfermedades productores La causa más frecuente de hipercalcemia en un pac iente ambulatorio
· Hiperuricosuria digestivas de ureasa es el hiperparatiroidismo primario. En cambio, la hipercalcemia más
· Hiperoxaluria · Ingesta excesiva frecuente en uno ingresado es la de origen neoplásico.
· Hipocitraturia de purinas
· Acidosis renal tubular · Fármacos
distal · litiasis úrica
No obstante, la mayoría de los casos de hiperoxaluria son secun·
· litiasis cálcica idiopática idiopática darios a malabsorción de ácidos grasos por enfermedades crónicas
pancreatobiliares, derivación intestinal para el tratamiento de la
Tabla 5. Tipos de litiasis. Situaciones que favorecen su aparición
obesidad mórbida, resección ileal, enfermedad inflamatoria intes·
tinal (MIR 05-06, 1 04; MIR 00-0 1 , 1 1 7), hipercalciuria coincidente
Edad temprana de aparición o por falta de calcio en la dieta, lo que permite que exista mayor
Litiasis bilateral cantidad de oxalato intestinal para su absorción.
litiasis en riñón único o malformado
La intoxicación por etilenglicol y metoxiflurano puede producir hi·
Composición poco frecuente
litiasis recidivante
peroxaluria, así como la ingesta de vitamina C en altas dosis. En
Nefrocalcinosis todos estos casos secundarios, el tratamiento con colestiramina, una
Litiasis coraliforme dieta pobre en grasas y la corrección de la malabsorción, en la me·
Tabla 6. Pacientes con indicación de estudio metabólico dida de lo posible, suelen ser medidas eficaces.
Hipocitraturia. Excreción de citrato inferior a 300 mgl24 h. Gene­
ralmente se asocia a otras anomalías urinarias. Aunque de causa
Desde el punto de vista práctico, las litiasis se pueden dividir en: las desconocida, puede contribuir una dieta rica en proteínas, hipoca­
de composición cálcica y las de otras composiciones, ya que el primer liemia, enfermedad intestinal o infección urinaria.
grupo supone la mayoría de los casos (70-80%) tratados habitualmente. Hiperparatiroidismo primario. Supone la causa más frecuente de
hipercalciuria conocida (véase Sección de Endocrinología, metabo·
lismo y nutrición) (MIR 98-99F, 1 44).
Litiasis cálcica Acidosis tubular renal distal (véase Sección de Nefrología). Enfer­
medad autosómica recesiva. Consiste en la imposibilidad del túbulo
En la mayoría de las ocasiones se desconoce el origen de la litiasis cál· distal para excretar hidrogeniones a la orina (orinas persistentemen­
cica, aunque se puede hacer una aproximación a los factores de riesgo te alcalinas) con aumento de la eliminación de calcio a la orina.
que influyen en su aparición. Sólo en un pequeño porcentaje de casos Existen formas incompletas que se observan en pacientes formado­
existe una enfermedad de base que puede ser tratada, y de esta forma res de cálculos de oxalato cálcico y con hipercalciuria idiopática.
desaparece la formación de cálculos cálcicos. En éstos, probablemente la acidosis tubular no juegue un papel im­
Hipercalciuria idiopática. Es la causa más frecuente de litiasis cál­ portante y responden a tiazidas (MIR 98-99, 1 38).
cica. Se define como una excreción urinaria de calcio mayor de Otras circunstancias que favorecen la litiasis cálcica son: sarcoido­
300 mgl24 h en el varón y 250 mgl24 h en la mujer. De cara a su sis, síndrome de Cushing, diuresis escasa, déficit de inhibidores o
manejo, las tiazidas disminuyen el calcio urinario, reduciendo la anomalías en el pH urinario (alcalosis).
formación de litiasis (MIR 03-04, 44; MIR 99-00F, 1 46; MIR 98- Litiasis cálcica idiopática. Aproximadamente en el 20% de los pa­
99, 1 35). La administración de citrato potásico ayuda a evitar la cientes con litiasis cálcica no se demuestra ninguna anomalía en el
hipopotasemia y aumenta el citrato urinario, que es inhibidor de la estudio metabólico.
litogénesis (Tabla 7).

I Litiasis úrica
ABSORTIVAS RESORTIVAS RENALES
,
Aporte excesivo Hiperparatiroidismo Acidosis tubular
El ácido úrico no disociado es poco soluble en orina. Con un pH uri­
·

Sd. de Surnen (leche · Inmovilización distal


y alcalinos ) • Tumorales Idiopátíca nario de 5, la solubilidad del ácido úrico es únicamente de 1 00 mgll,
Hipervitamlnosls O Enf. Paget mientras que con un pH de 7 es de 1 .580 mgll. Esto demuestra la gran
Idiopática · Sd. de Cushing importancia del pH urinario en la formación de cálculos de ácido úri­
Sarcoidosis
co. Aparte de éstos, también existe una pequeña proporción de cálcu­
Tabla 7. Causas más frecuentes de hipercakiuria los de urato monosódico y urato amónico.

Hiperuricosuria. Excreción en orina de más d e 800 mgl24 h en el El objetivo del tratamiento es reducir el ácido úrico excretado y au­
varón o 750 mgl24 h en la mujer. Además de favorecer la litiasis mentar el pH urinario (MI R 03-04, 80), ya que los cálculos más fre­
úrica, la hiperuricosuria constituye un factor de riesgo para la for­ cuentes en pacientes hiperuricémicos son los de ácido úrico. Por otra
mación de cálculos de calcio, probablemente por nucJeación hete­ parle, este tipo de cálculos son los que mejor responden al tratamien­
rogénea sobre núcleos de ácido úrico o urato sódico. Generalmente to médico mediante quimiólisis por alcalin ización urinaria. Pueden
se debe a un exceso de purinas en la dieta. administrarse diversos álcalis; el citrato potásico impediría el teóri­
Hiperoxaluria. Se considera como tal la excreción en orina de más co riesgo de formación de cálculos cálcicos por su efecto inhi bidor,
de 40 mgl24 h. Existe una hiperoxaluria primaria, que es conse­ pero también pueden tratarse con bicarbonato sódico o citrato sódi­
cuencia de un defecto enzimático autosómico recesivo; no tiene CO (MIR 02-03, 1 76). Una a lternativa es la acetazolamida en dosis de
tratamienlo y generalmente conduce a insuficiencia renal por litiasis 250 mgldía. Cuando, además, la uricemia es alta, puede tratarse con
recidivante. El único tratamiento que existe actualmente es el tras- alopurinol (MIR 06-07, 93).

12
urolOgia a
Litiasis cistínica complejo enzima-inhibidor irreversible. Se utilizan básicamente dos
sustancias de esta naturaleza: el ácido propiónico y el acetohidroxámi­
La cistinuria es un trastorno autosómico recesivo en el que existe un co. Su empleo suele venir acompañado de cefaleas, temblores, trom­
defecto de absorción, a nivel intestinal y tubular proximal, de los ami­ bosis venosas u otros síntomas neurológicos, por lo que tampoco son
noácidos dibásicos: cistina, ornitina, lisina y arginina (COLA), aunque de gran aceptación.
parece que puede existir un trastorno en el que únicamente se ve afec­
tada la cistina, lo que i ndicaría que, además de un mecanismo de trans­ Todo lo relativo al estudio de la nefrolitiasis expuesto anteriormente se
porte común, existe uno independiente para la cistina. puede repasar en la Tabla 8.

Los niveles de cistina en orina de 24 horas son superiores a 1 00 mg, de


hecho, los homoeigotos pueden excretar más de 600 mgldía. El diag­ Tratamiento de la litiasis ya formada (Figura 7)
nóstico se realiza identificando los característicos cristales hexagonales
en orina, o por una prueba positiva de nitroprusiato sódico (la orina se
tiñe de azul en pacientes afectados por esta enfermedad: test de Brand). Los cálculos ya formados no expulsables (> 4-5 mm) precisan de
tratamiento "agresivo", es decir, necesitan ser extraídos qui rúrgica­
El tratamiento consiste en aumentar la diuresis diaria (más de 3 I/día), mente o fragmentados de forma que puedan ser expu lsados espon­
alcalinizar la orina por encima de 7,5 y, en caso de que esto sea insufi­ táneamente.
ciente, puede iniciarse tratamiento con D-penicilamina (250 mgl6 h) o
a-mereaptopropionilglieina (250 mgl6 h). A continuación se analizan brevemente las diversas formas de tratamiento.
Cirugía. Ha sido el tratamiento estándar hasta la aparición de la
litotricia extracorpórea. Aún hoy día, es preciso recurrir a la cirugía
Litiasis infectiva cuando fracasan las ondas de choque o en determinados casos para
reducir la masa litiásica (cálculos coraliformes).
Los cálculos infectivos de estruvita o de fosfato amónico magnésico (Mg­ Endourología. La manipulación endoscópica de la vía urinaria es
NHl04-6H20) se desarrollan en un ambiente alcalino, producido por cada día más accesible gracias a las mejoras técnicas. Puede reali­
infección persistente de gérmenes que hidrolizan la urea, aumentando zarse extracción directa del cálculo mediante diversos tipos de pin­
la cantidad de amonio urinario. Los principales gérmenes que poseen zas o cestillas, o bien fragmentar previamente el cálculo mediante
ureasa, además de diversas especies de Proteus (MIR 06-07, 1 06). son diversas fuentes de energía, como la electrohidráulica, ultrasónica o
Pseudomonas, Klebsiella, Serratia y Enlerobacter. La presencia de cuer­ láser. Se puede acceder hasta el cálculo mediante ureterorrenosco­
pos extraños (sondas vesicales, suturas) favorece su formación. pia (U RS) o nefrolitotomía pereutánea ( NLPC).
litotricia extracorpórea por ondas de choque (LEOC). Las ondas de
Para su tratamiento se han empleado diversos métodos, generalmente choque se transmiten a través de los tej idos corporales con la misma
ineficaces. La antibioterapia únicamente mantiene estéril la orina du­ impedancia acústica que el agua hasta alcanzar la litiasis, sobre la
rante los cursos de tratamiento. que produce fenómenos de compresión y descompresión que con­
ducirán a su fragmentación.
Parece más prometedor el uso de inhibidores de la ureasa con ácidos Prácticamente todos los cálculos son susceptibles de tratamiento me­
hidroxámicos. Estos son moléculas análogas a la urea que forman un diante LEOe. La única limitación serían aquellos cálculos no localiza-

ESTRUVITA (FOSFATO
SALES CALCICAS AClDO URICO CISTlNA
AMÓNICO MAGNÉSICO)

. Oxalato cálcico: 55-60% 10-15% 5-10% 1 -3%


Frecuencia
Fosfato cálcico: 1 0-' 5%

Sexo Varón Mujer Varón Varón = Mujer

Hipercalciuria idiopática Infección por gérmenes ureasa (+) Gota (50%) Cistinuria
Etiologia Idiopática . Idiopática « 50%)
Hiperuricosuria (20%) . Hiperuricemias secundarias

pH Alcalino Alcalino Ácido Ácido

Radiologia Radioopacos Radioopacos Radiotransparentes Rad iolúcidos


Formas prismáticas polim6rficas Aglomerados de cristales Cristales hexagonales en prismas

:liD
Cristales-en ataúd- desorganizados, a veces formando o láminas
masas continuas

��

q�� &o�� 0
W
OO
Cristales de fostato Ca
Moñología

O rfF
de los cristales

� 1lJ
1Iil' -ft>
�� �
D � fSJ
Cristales de OxCa Cristales de estruvita Cristales de ácido úrico Cristales de cistina

Hipercalciuria idiopática: tiazidas Ácido propi6nico y ácido Alcalinizar la orina Forzar diuresis (ingesta hídrical
Hiperoxaluria 1 .11; piridoxina acetohidroxámico Alopurinol (si hay hiperuricemia) Alcalinizar orina
Tratamiento
Hiperoxaluria 2.11; colestiramina Antibioterapia Dieta de bajo contenido D-penicilamina (si no hay
En ocasiones cirugía proteico respuesta)

Tabla 8. Tabla-resumen de las nefrolitiasis

13
Manual CTO de Medicina y Cirugía, s.a edición

de su tamaño, composición y
dureza, localización, particula­
MANEJO DE LA UROLlTIASIS
ridades anatómicas de la vía ex­
cretora y paciente, función renal
y tipo de litotriptor disponible.
Crisis Estable
CUADRO AGUDO CUADRO CRÓNICO
La presencia de hipertensión

/ �
¿Edad?
Indican o arterial no controlada facilita
¿Tipo de cálculos?
no LEOC o
cirugía
¿Periodicidad de la clínica? el riesgo de hemorragia duran­
� �
No � �
com�licado - �� � � --
Complicado ¿Tipo de síntomas?
te la sesión de litotricia, luego
¿Viabilidad renal?
deberá ser estabil izada previa­
TTO. SINTOMATICO Obstrucción gr ave
mente a la misma y constituye,
Tratar la condición
·

Espasmolíticos · Infección, fiebre litotricia Cirugía en c ierto modo, por ello, con­
preexistente
y antiinflamatorios Dolor incoercible
t t t
·
traindicación relativa de LEOC
Reposo e hidratación · Riñón único

(MIR 08-09, 93; M I R 99-00F,


CALCICA EXTRACORPÓREA · URETEROTOMIA 1 43 ) (Tabla 9).
acidificar (lEOCJ · PIELOllTOTOMIA
Ecografía renal
(no útil si oxalato), PERCUTANEA • NEFRECTOMIA

citratos, tiazidas ultrasonográfica


colestiramina con microlumbotomía Complicaciones
Dilatación
y dieta baja en ENOOSCÓPICA
INGRESO YTTO. AGRESIVO grasas y rica vía ureteral
DRENAJE en calcio, si La expulsión de fragmentos Ii­
(catéter o nefrostomía) hiperoxaluria CONTRAINDICADA EN tiásicos puede ocasionar un
no. PARENTERAL ÚRICA · EMBARAZO

alcalinizar,
cólico renal y, con menor fre­
Antibióticos · INFECCIÓN
Remontar hemodinámica alopu rino l · OBSTRUCCIÓN DISTAL cuencia, obstrucción ureteral
Equilibrio electrolítico ESTRUVITA Aneurismas (sleinstrasse o "calle litiásica/l).
acetohid roxá m ico
·

Narcóticos · Coagulopatfas
VIGILANCIA ESTRECHA CISTINA Obesidad
O-penicilamina,
·

· Arritmia cardíaca Esta posib i l idad es mayor ante


vit. B� y alcalinizar litiasis de gran tamaño, por lo
que en algunos de estos casos
Figura 7. Manejo de la litiasis renal ya formada se puede colocar un catéter
de derivación urinaria (ne­
frostomía o doble J) antes de
bIes por su pequeño tamaño « 2-5 mm). Cualquier litiasis podría ser la LEOC para d i s m i n u i r este riesgo, generalmente en litiasis supe­
tratada con LEOC, aunque esto tendrá que ser matizado en función riores a 2 c m .

RElATIVAS (PRECISAN DE CONTROL PREVIO


Derivadas del efecto directo de las ondas de choque, pueden aparecer
ABSOLUTAS
AL TRATAMIENTO) contusiones renales manifestadas como hematuria, hematomas rena­
Embarazo Alteraciones de la coagulación
les, equimosis o eritema cutáneo, y en grado máximo, rotura renal. La
Obstrucción distal Aneurisma aórtico hematuria se considera la complicación más frecuente de la litotricia.
Infección activa Alteraciones del ritmo cardíaco, marcapasos
o desfibriladores
Más controvertida es la teórica relación de la LEOC con la aparición de
Obesidad
Hipertensión arterial descontrolada hipertensión arterial, ya que no está demostrada en las últimas revisiones
publ icadas, aunque sí la relación entre hematoma renal post-LEOC e
Tabla 9. Contraindicaciones de la LEOC (MIR 99-00F, 143) hipertensión arterial.

A un hombre de 29 años, con antecedentes de dolor tipo cólico en fosa renal izquierda días de evolución, asociado en las últimas 24 horas a fiebre, escalofríos y malestar
que cedió con tratamiento analgésico, se le practica una urografía intravenosa, apre­ general. Analítica de sangre: plaquetopenia, leucocitosis y disminución de la activi·
ciándose defecto de repleción radiotransparente de 6x7 mm en tercio distal de uréter dad de la protrombina. Analítica de orina normal. Radiografía de abdomen con claras
izquierdo. El pH de la orina fue de 5,5; asimismo, se observan cristales de urato, 9-12 imágenes de litiasis. Eco renal: dilatación moderada de sistema excretor izquierdo.
hematíes por campo y escasa leucocituria. ¿Cuál sería el tratamiento más apropiadol ¿Cuál es la conducta más adecuada?

1 ) Alopurinol vía oral. 1 ) Solicitar hemocultivos y urocultivo para establecer la necesidad de antibiote­
2) Ureteroscopia con extracción del cálculo. ra p i a.
3) Nefrolitotomía endosc6pica percutánea. 2) Realizar urografía intravenosa para intentar filiar la causa.
4) Alcalinización de la orina por vía oral. 3) Hidratar a la paciente bajo observación rigurosa, y repetir ecografía a l as 48
5) Administración de D-penicilamina. horas.
4) Colocar catéter doble) o practicar nefrostomía percutánea de forma inmediata
MIR 03-04, 80; RC: 4 con cobertura antibiótica.
5) laparotomía exploradora para objetivar causa, y realizar tratamiento antes de que
el cuadro esté muy evolucionado.
Mujer de 50 años, diabética insulinodependiente, con infecciones urinarias y cólicos
nefríticos de repetición. Acude a Urgencias por dolor en fosa renal izquierda de cinco RC: 4

14
04.
TUMORES RENALES

OnentaClón Aspectos esenciales

MIR
El aclenocarcinoma fenal es un
tema "de moda" en el examen (jJ El más frecuente de los tumores sólidos renales es el hipernefroma.
MIR. Cualquier aspecto de
este lema puede aparecer,
pero es fundamental reconocer
lIJ El paciente característico es un varón de mediana edad, obeso y fumador.
el síndrome de $Iauffer,
que ha sido recientemente GJ la tríada clásica consiste en hematuria, dolor y masa en flanco, actualmente, lo más habitual es que sea
incidentaloma (asintomático). Si produce síntomas, el más frecuente es la hematuria.
introducido y preguntado
varias veces consecutivas. Hay que sospechar tumor renal ante un varicocele izquierdo, de aparición súbita y que no cede con el
decúbito.

El hipernefroma puede producir multitud de síndromes paraneoplásicos. Esto puede complicar bastante el
diagnóstico, de ahí el sobrenombre de "tumor del internista".

La elevación de las transaminasas sin afectación hepática es típica del hipernefroma (síndrome de Stauffer).

No se debe confundir un quiste simple con un hipernefroma. los criterios de quiste simple son: contorno
liso, contenido transónico y refuerzo posterior.

la primera prueba de imagen, ante la sospecha de hipernefroma, sería la ecografía.

El tratamiento fundamental del hipernefroma es la extirpación quirúrgica. La quimioterapia y radioterapia


tienen un papel muy secundario.

4.1 . Ca rc inoma de cé l u las rena les


(adenoca rc inoma re nal, h ipernefroma)

Es e l tumor sólido renal más frecuente (90%) (Figura 8) (MIR 99-00, 1 77). Es un tumor fundamentalmente d e l a
edad adulta, con mayor incidencia entre los 4 0 y 60 años, con predominio e n e l varón 2 : 1 a excepción d e la
variedad cromófoba, típica de las mujeres.

Entre los factores de riesgo que se han implicado se encuentran el humo del tabaco, el cadmio y la obesidad.
Existen formas familiares que suelen ser múltiples y bilaterales, como en la enfermedad de Von Hippel·lindau y,
en menor medida, la esclerosis tuberosa (MIR 00·01 , 1 20). Se han identificado a lteraciones cromosómicas que
implican al cromosoma 3.

D RECUERDA Asimismo, existe una incidencia aumentada en el riñón poliquístico,


en la enfermedad quística adquirida de la insuficiencia renal crónica
la esclerosis tuberosa y la enferme·
dad Von Hippel.lindau se asocian y en los riñones malformados, como el riñón "en herradura". Procede
también a otro tumor renal, el an· de las células del túbulo contorneado proximal, y microscópica mente
IIJ Preguntas giomiolipoma.
predominan las células claras sobre las granulares y fusiformes .

• MIR 09-10, 99
- MIR 06-07, 102
- MIR 05-06, 105 Presentación
- MIR 04-05, 105
- MIR 01-02, 109
- MIR 00-0 1 , ' 2 0
- M I R 99-00, 1 7 7
- MIR 99-00F, 144
La tríada clásica: hematuria¡ dolor y masa en el flanco ocurre únicamente en el 1 0% de los casos y, cuando se
- MIR 98-99F, 146 presenta así, generalmente se trata de una enfermedad avanzada.

15
Manual eTO de Medicina y Cirugía, B.a edición

de su tamaño, composición y
dureza, localización, particula­
MANEJO DE LA UROllTIASIS
ridades anatómicas de la vía ex­
cretora y paciente, función renal
y tipo de l i lotriptor disponible.
Crisis Estable
CUADRO AGUDO CUADRO CRÓNICO
La presencia de h ipertensión

/ �
¿Edad?
Indican o arterial no controlada facilita
¿Tipo de cálculos?
no LEO( o
¿Periodicidad de la clínica? el riesgo de hemorragia duran­
-� � � - cirugía
Compl icado ¿Tipo de slntomas?
te la sesión de litotricia, luego
t
¿Viabilidad renal?
deberá ser estabil izada previa­
nO. SINTOMATICO Obstrucción grave
mente a la misma y constituye,
Tratar la cond ición
·

Espasmolfticos . Infección, fiebre litotricia Cirugía en cierto modo, por ello, con­
p reexistente
y a ntiinflamatorios Dolor incoercible
t t t
·
traindicación relativa de LEOC
Reposo e hidratación · Riñón único
(MIR 08-09, 93; M I R 99-00F,
CALCICA EXTRACORPÓREA · URETEROTOMIA 1 43) (Tabla 9).
acidificar (LEOCI · PIELOLITOTOMIA
Ecografía renal
(no útil si oxalato), PERCUTANEA · NEfRECTOMIA

citratos, tiazidas ultrasonográfica


colestiramina con microlumbotomía Complicaciones
Dilatación y dieta baja en ENOOSCÓPICA

INGRESO VITO. AGRESIVO grasas y rica vía ureteral


DRENAJE
en calcio, si La expulsión de fragmentos Ii­
(catéter o nefrostomía) hi peroxal uria CONTRAINDICADA EN tiásicos puede ocasionar un
ÚRICA EMBARAZO
no. PARENTERAL ·
cólico renal y, con menor fre­
Antibióticos alcali nizar, INFECCIÓN
cuencia, obstrucción ureteral
·

Remontar hemodinámica alopurinol · OBSTRUCCIÓN DISTAL


ESTRUVITA Aneurismas (steinstrasse o "calle litiásica").
Equi librio electrolítico ·

Narcóticos acetohidroxámico · Coagulopatfas


VIGILANCIA ESTRECHA CISTlNA Obesidad
Esta posibilidad es mayor ante
·

D-penici lam i na, · Arritmia cardiaca


vit. B6 y alcalinizar litiasis de gran tamaño, por lo
que en algunos de estos casos
Fig ura 7. Manejo de la litiasis renal ya formada
se puede colocar un catéter
de derivación urinaria (ne­
froslomía o doble J) antes de
bies por su pequeño tamaño « 2-5 mm). Cualquier litiasis podría ser la LEOC para dismi n u i r este riesgo, generalmente en litiasis supe­
tratada con LEOC, aunque esto tendrá que ser matizado en función riores a 2 cm.

RelATIVAS (PRECISAN DE CONTROL PREVIO


Derivadas del efecto directo de las ondas de choque, pueden aparecer
ABSOLUTAS
Al TRATAMIENTO) contusiones renales manifestadas como hematuria, hematomas rena­

Embarazo
les, equimosis O eritema cutáneo, y en grado máximo, rotura renal. La
Alteraciones de la coaguladón
Obstrucción distal Aneurisma aórtico hematuria se considera la complicación más frecuente de la litotricia.
I nfección activa Alteraciones del ritmo cardiaco, marca pasos
° desfibriladores
Más controvertida es la teórica relación de la LEOC con la aparición de
Obesidad
Hipertensión arterial descontrolada
hipertensión arterial, ya que no está demostrada en las últimas revisiones
publicadas, aunque sí la relación entre hematoma renal post-LEOC e
Tabla 9. Contraindicaciones de la LEOC (MIR 99-OOF, 143) hipertensión arterial.

A un hombre de 29 años, con antecedentes de dolor tipo cólico en fosa renal izquierda días de evolución, asociado en las úllimas 24 horas a fiebre, escalofríos y malestar
que cedió con tratamiento analgésico, se le practica una urografía intravenosa, apre.­ general. Analítica de sangre: plaquetopenia, leucocitosis y disminución de la activi·
ciándose defecto de repleción radiotransparenle de 6x7 mm en tercio distal de uréter dad de la protrombina. Analítica de orina normal. Radiografia de abdomen con daras
izquierdo. El pH de la orina fue de 5,5; asimismo, se observan cristales de urato, 9-12 imágenes de litiasis. Eco renal: dilatación moderada de sistema excretor izquierdo.
hematíes por campo y escasa leucocituria. ¿Cuál sería el tratamiento más apropiado? ¿Cuál es la conducta más adecuadal

1 ) Al opurinol vía ora l. 1 ) Solicitar hemocu lti vos y urocultivo para cstablecer la necesidad de antibiotc­
2) Ureleroscopia con exlracción del cálculo. rapia.
3) Ncfrolitolomía endoscópica percutánea. 2) Realizar urografía intravenosa para intentar filiar la causa.
4) A lca lini z¡lción de la orina por vía oral . 3) Hidratar a la paciente baj o observación rigurosa, y repetir ecografía a las 48
5) Admin ist ración de D-penicilamina. horas.
4) Colocar catéter doble J o practicar nefrostomfa percutánca de forma inmediata
MIR 03·04, 80; RC: 4 con cobertura antibiótica.
5) laparotomía exploradora pJra objetivar causa, y re;¡lizar tratamiento antes de que
el cuadro esté muy evolucionado.
Mujer de 50 años, diabética insulinodependiente, con infecciones urinarias y cólicos
nefríticos de repetición. Acude a Urgencias por dolor en fosa renal izquierda de cinco RC: 4

14
04.
TUMORES RENALES

Orll?ntaclón Aspectos esenciales

MIR
El adenocarcinoma renal es un
tema "de moda� en el examen El más frecuente de los tumores sólidos renales es el hipernefroma.
MIR. Cualquier aspecto de
este lema puede aparecer, El paciente característico es un varón de mediana edad, obeso y fumador.
pero es fundamental reconocer
el sindrome de Stauffer, la tríada clásica consiste en hematuria, dolor y masa en flanco, actualmente, lo más habitual es que sea
que ha sido recientemente
incidentaloma (asintomático). Si produce síntomas, el más frecuente es la hematuria.
introducido y preguntado
varias veces consecutivas. Hay que sospechar tumor renal ante un varicocele izquierdo, de aparición súbita y que no cede con el
decúbito.

El hipernefroma puede producir multitud de síndromes paraneoplásicos. Esto puede complicar bastante el
diagnóstico, de ahí el sobrenombre de "tumor del internista".

la elevación de las transaminasas sin afectación hepática es típica del hipernefroma (síndrome de Stauffer).

No se debe confundir un quiste simple con un hipernefroma. los criterios de quiste simple son: contorno
liso, contenido transónico y refuerzo posterior.

la primera prueba de imagen, ante la sospecha de hipernefroma, sería la ecografía.

El tratamiento fundamental del hipernefroma es la extirpación quirúrgica. la quimioterapia y radioterapia


tienen un papel muy secundario.

4.1 . Ca rci noma de c é l u l a s ren a l es


(adenocarc i n oma ren a l, h i pernefroma)

Es e l tumor sólido renal más frecuente (90%) (Figura 8) (MIR 99-00, 1 77). Es un tumor fundamentalmente d e l a
edad adulta, con mayor incidencia entre los 40 y 60 años, con predominio e n e l varón 2 : 1 a excepción d e la
variedad cromófoba, t ípi ca de las mujeres.

Entre los factores de riesgo que se han impl icado se encuentran el humo del tabaco, el cadmio y la obesidad.
Existen formas familiares que suelen ser múltiples y bilaterales, como en la enfermedad de Von Hippel -Li ndau y,
en menor medida, la esclerosis tuberosa (MIR 00-01 , 1 20). Se han identificado alteraciones cromosómicas que
implican al cromosoma 3 .

D RECUERDA Asimismo, existe una incidencia aumentada en el riñón poliquístico,


en la enfermedad quística adquirida de la insuficiencia renal crónica
la esclerosis tuberosa y la enferme­
dad Von Hippel-lindau se asocian y en los riñones malformados, como el riñón "en herradura". Procede
también a otro tumor renal, el an­ de las células del túbulo contorneado proximal, y microscópicamente
III Preguntas giomiolipoma.
predominan las células claras sobre las granulares y fusiformes .

• MIR 09-1 0, 99
- MIR 06-07, 102
. MIR 05·06, 105 Presentación
. MIR 04-05, 105
- MIR 01-02, 109
- MIR OO-Ol, 1 20
- MIR 99-00, 177
La tríada clásica: hematuria, dolor y masa en el flanco ocurre ú nicamente en el 1 0% de los casos y, cuando se
- MIR 99-00F, 144
- MIR 98-99F, 146 presenta así, generalmente se trata de una enfermedad avanzada.

15
Manual (TO de Medicina y Cirugía, 8.a edición

Figura 9. Ecografía de quistes renales simples

la realización de punción-aspiración con aguja fina (PAAF) de una


masa renal para su filiación es una exploración agresiva que, debido
Figura 8. Carcinoma de células renales
a su baja sensibilidad, no se justifica actualmente, excepto en casos
excepcionales.
El 30% presenta metástasis a distancia en el momento del diagnóstico,
aunque contrariamente, cada vez son más los hallazgos incidentales al la tomografía axial computarizada (fe) es el mejor método aislado para
realizar ecografías abdominales de rutina por otra causa, alcanzando evaluar una masa renal, proporcionando información precisa sobre metás­
en algunos estudios más de la mitad de los casos diagnosticados. tasis ganglionares (80%) y afectación de órganos adyacentes (Figura 1 0).

la anomalía más frecuente es la hematuria macroscópica o micros­


cópica (60%). Otros hallazgos frecuentes son dolor (40%), pérdida de
peso (30%), anemia (40%), masa en flanco (24%), HTA (20%), hiper­
calcemia (6%), eritrocitosis (3%). El 20% de los pacientes presentan
como cuadro paraneoplásico alteración de las enzimas hepáticas sin
evidencia de metástasis (síndrome de Stauffer) (MIR 09- 1 0, 99; MIR 06-
07, 1 02 ; M I R 05-06, 1 05).

Ocasionalmente, el adenocarcinoma renal puede producir hormonas


productoras de síndromes clínicos según la sustancia secretada. Entre
éstas se encuentran péptidos PTH-like, prostaglandinas, prolactina, re­
nina, gonadotropinas o corticoides. la invasión de la vena renal princi­
pal en el lado izquierdo puede ocasionar la aparición de un varicocele
de forma repentina, que no disminuye en decúbito.

o RECUERDA
la producción de péptidos puede aparecer en el hipernefroma, pero es
más típica de carcinomas epidermoides (pulmón, esófago, etc.).

"

"

� Diag nóstico
o
E
.=
La ecografía es la primera prueba complementaria que debe realizarse, La RM, aunque no se emplea de manera rutinaria en este tipo de pa­
de forma que la identificación ecográfica de una lesión que cumple cri­ cientes, sí se utiliza como método de diagnóstico básico en sujetos en
terios de quiste simple (contorno liso, contenido transónico y refuerzo los que se sospecha afectación lrombótica tumoral de la vena renal o
posterior) hace innecesarios mayores esfuerzos diagnósticos, pudiendo cava (MIR 04-05, 1 05) (Figura 1 1 ).
hacerse un seguimiento ecográfico anual (MIR 99-00F, 1 44). De esta
forma se diagnostican l a mayoría de las masas renales en la actualidad Aunque la urografía intravenosa (U IV) continúa siendo la base del diag­
(Figura 9). nóstico por imagen en urología, en el caso del adenocarcinoma renal

16
UrOIOgía �

proporciona pocos datos y de forma indirecta, como puede ser la dis­ Trata miento
torsión del sistema colector, su ocupación o la anulación funcional del
riñón. En las placas tomográficas de la UIV puede observarse la presen­ Una vez estudiado el tumor y descartada la presencia de metástasis,
cia de una masa o una alteración del contorno renal. tanto viscerales como ganglionares, el tratamiento de elección es la
nefrectomía radical, incluyendo la fascia de Gerota y la glándula su­
prarrenal (MIR 01 -02, 1 09).

En ocasiones especiales se puede plantear la cirugía conservadora o


parcial, como en los tumores bilaterales, en aquellos que aparecen so­
bre riñón único, o sujetos con nefropatías médicas, en los que la pérdi­
da de masa nefronal obligaría a diálisis.

Además de en estas indicaciones imperativas, actualmente es el nuevo


patrón de referencia en el tratamiento de tumores pequeños (menos de
4 cm), bien delimitados y sin afectación de la grasa perirrenal. En estos
pacientes seleccionados parece que la supervivencia y la tasa de reci­
divas locales son semejantes a las que se presentan en casos similares
tratados con nefrectomía radical.

La linfadenectomía regional no mejora la supervivencia y únicamente


tiene, por tanto, validez en la estadificación, por lo que no se realiza
sistemáticamente. É ste se establece sobre la base de los hallazgos qui­
rúrgicos y anatomopatológicos.
Figura 1 1 . R M de tumor renal con trombo en venas renal '1 cava

Otras formas de tratamiento carecen de eficacia. Tanto la quimiotera­


La arteriografía renal, exploración obligada hace años, ha quedado re­ pia como la radioterapia ofrecen resultados pobres. En el caso de en­
legada a los casos dudosos, riñones únicos y otras situaciones en las fermedad metastásica, las opciones son múltiples, pero ninguna satis­
que se plantea tratamiento quirúrgico conservador. El patrón arteriográ­ factoria. Aunque se ha descrito la regresión de las lesiones metastásicas
fico característico incluye neovascularización tumoral, lagos venosos, tras la nefrectomía, esto ocurre únicamente en u n 1 %, Y generalmente
fístulas arteriovenosas y vasos capsulares. de forma transitoria, por lo que no se justifica, salvo de forma paliativa
por otros motivos.
El estudio de extensión, si se sospechan metástasis, se completará rea­
lizando radiografía de tórax, analítica hepática completa y, en algunos La inmunoterapia con i nterferones, interleucina, l infocitos killer acti­
casos dudosos, gammagrafía ósea. vados, y ciertas combinaciones de quimioterapia con inmunoterapia,
son alternativas para la enfermedad metastásica, pero en ninguna de
El procedimiento diagnóstico ante la presencia de masas renales se ellas se obtienen tasas de respuesta superiores al 1 5%. Actualmente
puede observar en la Figura 1 2. se utilizan de preferencia: antitirosincinasas en primera línea, an­
tiangiogénicos en segunda línea. Las situaciones que favorecen l a
respuesta al tratamiento i n m unoterapéutico son: presencia d e me­
tástasis pulmonares exclusivamente, buen estado general, y que se
MASA RENAL descubierta accidentalmente
haya realizado la nefrectomía a n tes del descubrimiento de las masas
I pul monares .
t
. Examen físico
. Análisis de orina
En la actualidad se i nvestiga sobre autovacunas elaboradas con linfoci­
tos peritumorales que parecen ofrecer resultados alentadores.

Quiste complejo
o masa sólida
Quiste simple
4.2. Otros tumores

Observar
Tumor d e Wilms (véase Sección de Pediatría ) .
Tumores renales metastásicos. Pueden encontrarse metástasis en el
riñón de tumores de pulmón (la más frecuente), mama, mela nomas
Masa sólida e infiltración por linfoma.
o quiste complicado Angiomiolipoma
Tumores benignos:

/ Adenomas corticales: son los tumores más frecuentes del adulto,


aunque indistinguibles clínicamente del adenocarcinoma, por lo
NEFRECTOMIA No complicado Complicado
que se tratan como tales. E l criterio clásico de tamaño (3 cm)
RADICAL OBSERVACiÓN · Nefrectomía simple
O PARCIAL · Nefrectomía parcial para su diagnóstico diferencial no es válido en la actualidad.
· Embolización Angiomiolipomas: se asocian a la esclerosis tuberosa en un 50%
(M I R 98-99F, 146). Compuestos de una proporción variable de
Figura 12. Algoritmo diagnóstico de las masas renales grasa, vasos y fibras musculares. Cuando son grandes (mayores

17
Manual ero de Medicina y Cirugía, B.a edición

de 4 cm), pueden ocasionar u n síndrome de Wünderlich por guirlo del adenocarcinoma, pero en la mayoría de los casos,
sangrado relroperitonea l . Cuando se asocian a esclerosis tubero­ ni éstos ni la citología o la biopsia, ofrecen garantías sufi­
sa, suelen ser múltiples y bilaterales, por lo que deben tratarse de cientes de su benignidad, por 10 que tienden a ser tratados
forma conservadora. mediante nefrectomía.
Oncocitoma: considerado benigno, aunque en algunos se han Nefroma mesoblástico (hamarloma fetal): es el tumor benigno
detectado metástasis. Hay criterios radiológicos para dislin- más frecuente en recién nacidos y lactantes.

Casos clínicos representativos

Un hombre de 45 años tiene un carcinoma de células renales extendido. los niveles Un paciente de 62 años, con alteración de la función renal y crisis de hematuria,
de GOl, fosfatasa alcalina, lOH y a-2 globulina son elevados y el tiempo de protom­ presenta una masa abdominal palpable en nanco derecho. Se le realiza una le, de­
bina alargado. El hígado aparece difusamente agrandado, pero no existen defectos tectándose una masa de carácter sólido de 8 cm de diámetro en riñón derecho. En la
focales de infiltración intrahepática. la explicación etiológica más probable para anamnesis destaca que el paciente es fumador de 35 cigarrillos al día. ¿Cuál es, entre
estos hallazgos será: los siguientes, el diagnóstico de presunción más probable?

1 ) los efectos hcpdlolóxicos de tumor. 1 ) Nefroblasloma.


2) Metásl,lSis hep.ltica. 2) Uposarcoma.
3) Amiloidosis. 3) Angiomiolipoma.
4) Trombosis tumorales que obstruyen la vena hepática. 4) Adenocarcinoma..
5) Hepatitis vírica aguda. 5) Carcinoma epidermoide.

MIR 05-06, 105; RC: 1 MIR 99-00, 1 77; RC: 4

18
05.
HIPERPLASIA y CARCINOMA
PROSTÁTICO

OrientacIón Aspectos esenciales

MIR
La hiperplasia prostática benigna (HPB) suele afectar a la zona periuretral de la glándula. El cáncer aparece
Tanto la hiperplasia
prostática benigna como
[jJ en la zona periférica.
el cáncer de próstata, son
dos temas fundamentales. La hiperplasia prostática benigna no guarda relación con el cáncer.
Probablemente el cáncer sea
más importante, sobre todo Tanto la HPB como el cáncer tienen relación con las hormonas sexuales, y suelen aparecer en varones
en lo referente al tratamiento. ancianos.
Hay que aprenderse muy bien
el resumen de la Tabla 1 1; El tratamiento médico de la HPB consiste en a-bloqueantes (relajan la musculatura uretral y del cuello
aporta muchas preguntas vesical), inhíbidores de la 5 a-reductasa (disminuye el tamaño glandular) y (¡toterapía. Esta última no ha
acertadas a cambio de poco demostrado utilidad con parámetros objetivos.
esfuerzo.
El tratamiento definitivo de la HPB es la cirugía, que puede consistir en resección transuretral o en cirugía
abierta, dependiendo del tamaño prostático.

El cáncer de próstata es casi siempre un adenocarcinoma, con gran frecuencia multifocal.

El cáncer de próstata cada vez se diagnostica con más frecuencia en fase asintomática. Cuando presenta
clínica, puede consistir en síntomas urinarios similares a la HPB.
El PSA elevado no es diagnóstico de cáncer de próstata. Puede corresponder a una HPB. El diagnóstico defi­
nitivo de cáncer prostático precisa una biopsia.

Las metástasis lumbares son típicas del cáncer de próstata, pudiendo producir compresión medular.

El tacto rectal revela una próstata pétrea e irregular en el cáncer de próstata. Sin embargo, al principio puede
no ser palpable, ni visible en la ecografía (T1).

La principal complicación quirúrgica del cáncer de próstata es la impotencia.

Ante un síndrome de compresión medular por cáncer de próstata, nunca se deben emplear análogos de la
LHRH únicamente. Siempre deben asociarse antiandrógenos.

En el cáncer de próstata, la indicación más clara de prostatectomía radical es el estadio T2a.

El tratamiento fundamental del cáncer de próstata diseminado es la hormonoterapia.

5 . 1 . H i perplasia prostática ben i g na

La hiperplasia prostática benigna (HPB) afecta en mayor o menor grado a la gran mayoría de los varones a partir
de la quinta década de la vida, alcanzando el 80-95% de la población masculina de 80 años.

La próstata se divide clásicamente en cinco lóbulos (anterior, medio, posterior y dos laterales); aunque éstos úni­
camente se encuentran como tales en la edad fetal. En el adulto se puede interpretar la anatomía de la próstata
dividida en dos partes: una zona periférica, donde se origina principalmente el carcinoma, y una zona periure­
iIl Preguntas tral o transicional, de la que procede la HPB (Figura 1 3) .

• MIR 08-09, 106


La HPB está compuesta de una proliferación variable de elementos glandulares, musculares y del estroma, que en
- MIR 06-07, 1 03, 233
- MIR 05-06, 106 su crecimiento comprimen la próstata periférica, formando la l l amada cápsula quirúrgica. Su etiopatogenia no está
- MIR 04-05, 1 06, 225 clara; aunque el estímulo androgénico a través de su forma activa, la dihidrotestosterona, es fundamental, su papel
- MIR 03-04, 91
- MIR 02-03, 188 exacto no ha sido determinado. Las teorías más recientes abogan por un desequil ibrio hormonal de estrógenos/an­
- MIR 01 -02, 1 04, l OS, 1 07 drógenos, o fX'r la existencia de factores de crecimiento prostáticos con un papel permisivo del ambiente hormonal.
- MIR aQ-Q1 , 1 1 9
- MIR 98-99, 1 3 7
- M I R 98-99F, 147 No existe evidencia de asociación entre HPB y carcinoma prostático.

19
Manual eTO de Medicina y Cirugía, B.a edición

Fase clínica (Figura 1 5). La elongación de las fibras musculares por


Estroma fibromuscular anterior
encima de un límite condiciona pérdida de capacidad contráctil. En

\?7
este momento aparece retraso del inicio de la micción, disminución
del calibre y de la fuerza del chorro micdonal y alargamiento del
vaciado (10 que en conjunto se denomina síndrome prostático), El va·
ciado suele ser incompleto, dando lugar a un residuo postmiccional.

�--=,- Zona periférica

� zona central

Glándulas
Lóbulo anterior suburetrales
Conductos
eyaculadores

Lóbulo
lateral

Lóbulo posterior

Figura 13 . Anatomra de la próstata


Urografía lntraveno� mostrando Impronta prostática en vejiga (vejiga ·en montera")
con uréteres en anzuelo

Diagnóstico Figura 1 S. Hiperplasia prostática en fase clínica

Fase de descompensación (Figura 1 6). Se produce un vencimiento


El crecimiento prostático generalmente se produce hacia la uretra, del detrusor vesical, que es incapaz de vencer la presión uretral, au­
ocasionando obstrucción de ésta y dificultando el vaciamiento vesical mentando la sintomatología anterior y pudiendo presentarse reten­
(Figura 1 4). Esto no se manifiesta inmediatamente, sino que, general­ ción urinaria. Ocasionalmente puede aparecer dilatación ureteral
mente, el proceso pasa por una serie de etapas que incluyen una fase bi lateral con deterioro de la función renal. Esto se debe a uropatía
de compensación, una fase clínica y una de descompensación. obstructiva infravesical con pérdida del mecanismo antirreflujo.

8
"
19
e"­

E
o
e
'2

u

.•
,

c.
"
"-
I Figura 14. Ecografía de hiperplasia prostática con crecimiento
del 16bulo medio intravesical Situación del mismo paciente un año después de la anterior: hidronefrosis grave.
De la vejiga (no se observa) se evacuaron 3500 ce de orina

Fase de compensación. El crecimiento prostático ocasiona un au­ Figura 16. Hiperplasia prostática en fase de descompensaci6n

mento de la presión uretral durante el vaciado que es compensado


por una mayor actividad contráctil del detrusor que se hipertrofia, Pueden producirse también otro tipo de síntomas denominados "irrita­
encontrando presiones vesicales más elevadas. En esta fase, la clíni­ tivos", que son debidos a la alteración funcional vesical, y cuya reso­
ca puede ser mínima o inexistente. lución es más difícil tras la desaparición de la obstrucción (MIR 01 -02,

20
u rolo 9ia a
1 04). Entre estos síntomas se incluyen polaquiuria, tenesmo, nicturia y doxazosina, terazosina, tamsulosina, etc.) que relajan la musculatura
urgencia mlccional. La HPB es la causa más frecuente de obstrucción del cuello vesical y uretra. Hasta ahora, estos fármacos se han estado
del tracto urinario inferior en el varón. utilizando en forma de escalera terapéutica, pero la aparición del es­
tudio COMBAT parece indicar que en pacientes con sintomatología a
En la evaluación del síndrome prostático, el tacto rectal continúa sien­ partir de moderada, y con volúmenes prostáticos por encima de 30-40
do la exploración fundamental, sobre todo para diferenciarlo del car­ ce se debe realizar terapia combinada de inicio.
cinoma, ya que no es infrecuente que ambas entidades coexistan. La
clínica es lo más importante para valorar la indicación de tratamiento
de la HPB, ya que no existe correlación entre el tamaño prostático y el
o RECUERDA
La finasterida también es útil para la alopecia androgéni ca, donde se
grado de obstrucción (MIR 08-09, 1 06). Cualquier zona sospechosa al emplea en dosis mucho menores.
tacto debe ser biopsiada.

La medición del flujo máximo miccional es también importante, conside­ Como inconvenientes principales de los inhibidores de la 5 a-reductasa,
rándose normal cuando es mayor de 1 5 mi/s y claramente patológico si se encuentran: i mpotencia, reducción del PSA en torno al 50% (difi­
es menor de 1 0 mi/s. El estudio puede completarse con una ecografía que cultando el diagnóstico del carcinoma, si lo hubiese) y que tarda una
permita evaluar si existe afectación del tracto urinario superior, residuo media de cuatro meses en hacer efecto.
postmiccional, litiasis vesical u otra patología asociada. El uso del PSA en
la HPB únicamente está indicado para descartar la presencia de carcino­ De los a-bloqueantes, el inconveniente principal es la hipotensión.
ma en la próstata, ya que no sirve para diagnosticar HPB, aunque recien­
temente ha demostrado ser el mejor predictor de la historia natural de la En cuanto a las indicaciones de tratamiento quirúrgico, globalmente,
enfermedad. Es decir, que mayores niveles de PSA en HPB diagnosticada sólo un 1 0% de los pacientes prostáticos precisará cirugía. La inten­
probablemente se correlacionarán con mayores volúmenes prostáticos y sidad de las manifestaciones clínicas subjetivas y la mala respuesta al
con más posibilidades de complicación derivadas de la HPB. tratamiento médico pueden constituir la indicación para la interven­
ción. Entre las causas "objetivas" que suponen indicación absoluta de
tratamiento quirúrgico se encuentran (MIR 01 -02, 1 05):
Trata miento Retención urinaria reiterada.
Hidronefrosis retrógrada (lesión del parénquima renal por obstruc­
ción infravesical).
Dentro de las posibilidades terapéuticas, la cirugía continúa siendo el Infección urinaria de repetición.
único tratamiento definitivo para la HPB. tsta puede ser endoscópica Litiasis vesical.
(RTUp: resección transuretral prostática) o abierta (adenomectomía pros­ Hematuria de repetición.
tática) (Figura 1 7), dependiendo del tamaño del adenoma. En el 1 0% de
las piezas obtenidas se encontrarán focos de adenocarcinoma incidental.

5.2. Carci noma p rostático

El adenocarcinoma prostático es el tumor maligno más frecuente del


aparato genitourinario masculino y el segundo en frecuencia general,
después del pulmonar. Sin embargo, si se incluyesen los carcinomas
incidentales y los encontrados en autopsia, supera al pulmonar en pre­
valencia (MIR 06-07, 1 03).

La hormonodependencia del cáncer prostático parece indicar el papel


de los andrógenos en su etiología o patogenia. La relación de factores
genéticos, ambientales o infecciosos no ha quedado suficientemente
establecida.

El 95% de los carcinomas prostáticos son adenocarcinomas originados


en la zona periférica de la próstata. Los carcinomas ductales se originan
en los conductos prostáticos en lugar de los acinos, e histológicamente
Figura 17. Adenomectomia prost áti ca
pueden corresponder a carcinomas transicionales, escamosos, endome­
trioides o mixtos. Más raros son los carcinosarcomas (menos del 1 %).
Se debe tener en cuenta que en la cirugía de la HPB no se extirpa la
cápsula quirúrgica, que está constituida por las glándulas prostáticas El adenocarcinoma prostático, con frecuencia, es multifocal y presenta
periféricas comprimidas por el adenoma, y es el principal origen del poblaciones en distinto grado de diferenciación. En esta heterogenei­
carcinoma prostático, por lo que la intervención quirúrgica no protege dad se basa la clasificación de Gleason, que asigna una puntuación de
del desarrollo de este proceso. 1 a 5, según el patrón histológico de cada una de las dos poblaciones
más representativas de la masa, sumando ambas puntuaciones para
Los tratamientos no quirúrgicos incluyen una variedad de fitoterapias, obtener un resultado final de 2 a 1 0. Esta escala de Gleason se corres­
poco efectivas si se valoran con parámetros objetivos, inhibidores de la ponde con el pronóstico de la enfermedad, independientemente del
5 a-reductasa (finasterida, dutasterida) (MIR 04-05, 225) que reducen el estadio (MIR 06-07, 233). Para la estadificación se emplea principal­
tamaño prostático, antagonistas a-adrenérgicos (alfuzosina, prazosín, mente la clasificación TNM (Tabla 1 0 y Figura 1 8) (MIR 98-99F, 1 4 7).

21
Manual eTO de Medicina y Ci rug ía, s.a edición

Cl ínica

. T1: tumor ¡naparente cI(nicamente (no pal pable ni visible por técnicas de
i magen) El carcinoma prostático es una enfermedad más frecuente en ancia­
T1 a: hallado incidentalmente. Afectación menor del 5% del tej ido resecado nos, y la mayoría de ellos se diagnostica por encima de los 60 años.
• T l b: hallado incidentalmente. Afectación mayor del 5% del tejido resecado
Clínicamente puede producir síntomas obstructivos del tracto urina­
• Tlc: Tumor identificado por p u nción-biopsia por aumento del PSA

T2: tumor confinado a la próstata (in cl uye la inva sión de la cápsula prostática rio inferior superponibles a los de la HPB. A éstos puede añadi rse la
sin exteriorización del tumor hacia el tejido adiposo periprostático) hematuria. El 25% de los pacienles que refieren retención urinaria
T2a: menos del 50% de u n lóbulo aguda presentan u n carcinoma prostático. Aproximadamente u n 25%
- T2b: más del 50% de un lóbulo
- T2c: dos l óbulos
de los pacientes presentan metástasis en el momento del diagnóstico;
T3: extensión del tumor por fuera de la cápsula éstas pueden producir man ifestaciones como dolor óseo, compresión
- T3a: extensión transcapsular (sea unilateral o bilateral) medular, mieloptisis o coagulopatía. Afortunadamente, estos casos
- T3b: invasión de la(s) veslc u l a (s) seminal{es)
se encuentran en claro descenso gracias a la i ncorporación del PSA
T4: tumor fijo o invade órganos adyacentes distintos a las vesículas
(prostate-specific a n tigen -a n tígen o prostático específico), facilitando
sem inales (cuello vesical, esfínter externo, recto, músculo elevador
o pared el diagnóstico de la enfermedad en estadios tempranos y comúnmen­
te asintomáticos.
NX: no se pueden estudiar los ganglios regionales
NO: no metástasis ganglionares
N 1: metástasis a Diagnóstico

MO: no metástasis
M l : metástasis a distancia Tacto rectal
Ml a: ganglios linfáticos no regionales
Mlb: h ueso
M 1 c: otras local izacion es Continúa siendo el método fundamental de cribado. Son accesibles al
tacto rectal todos los estadios excepto el T1, que por definición es un
Tabla 1 0. Estadificación del carcinoma de próstata
hallazgo. Característicamente, el carcinoma es duro, nodular e irre·

T: tumor primario cUnlca TN


na

Tla Tlb n
TX NO puede evaluar el tumor
TO No existen signos de tumor primario

na nb

Tlc nb

<5% >5%

T1 Tumor no evidente dlnic.amente, no palpable T2 Tumor limitado a la próstata o a la T3 Tumor que se extiende a través de la
ni visible mediante técnicas de imagen: capsula, sin sobrepasarla: capsula prostática:
na Extensión menor o igual a1 5% del tejido · T2a Menos del 50% de un lóbulo . TIa Extensión extracapsular (unilateral
resecado · T2b Más del 50% de un lóbulo o bilateral)
. Tl b Extensión mayor del 5% del tejido · T2c Dos lóbulos . TIb Tumor invade la veslcula seminal
resecado
. TlcTumor identificado mediante punción
biópsica (consecuenc.ia de un PSA elevado)
N: ganglios linfáticos regionales

T4 Tumor fijo o que invade estructuras adyacentes diferentes a las veslculas seminales NX No se pueden evaluar los ganglios linfáticos regionales
NO No hay metástasis ganglionares regionales
Nl Metástasis en ganglios linfáticos regionales

Figura 18. Estadificación del adenoca rd noma de próstata

22
Ur0109¡a .a
guiar. En general, se aconseja un tacto rectal y un PSA anual a todos tivo se ha i ntentado aumentar su especificidad para cáncer con otros
los varones por encima de 50 años aunque, de momento, la OMS no parámetros (densidad de PSA, índice PSNedad, velocidad de cambio
aconseja la realización de cribado poblacional sistemático. del PSA, PSA libre), aunque aún no ha quedado establecida la ventaja
de éstos sobre el PSA aislado.

Marcadores tumorales Si el PSA es menor de 4 ng/ml, es poco probable que se encuentre un


cáncer de próstata. Si es mayor de 1 0, las probabi lidades aumentan, lo
Se dispone fundamentalmente de dos marcadores tumorales. La fosfa� que aconsejaría una biopsia de próstata ecodirigida. S i está entre 4 y
tasa ácida prostática (FAP) se emplea en clínica desde hace décadas, 1 0, se pueden utilizar los parámetros antes mencionados para valorar
es un marcador específico, pero su elevación suele indicar extensión la necesidad de biopsia (MIR 0 1 -02, 1 07).
extraprostática, por lo que no resulta útil en el diagnóstico precoz. El
antígeno prostático específico (PSA) es realmente un marcador de te�
jido prostático cuyos niveles suelen encontrarse más elevados en el Pruebas de imagen
cáncer, pero es inespecífico y también están elevados a consecuencia
de patología benigna (infecciones, sondajes, HPB, etc.). Por este mo� La ecograría transrectal (ETR) (Figura 1 9) es el método de imagen más
útil para la esladificación local, pudiendo ofrecer información impor­
tante sobre la afectación capsular, de vesículas seminales, cuello vesi­
cal o recto. Aunque no existe un patrón característico, suele aparecer
como nódulos hipoecogénicos. La ecografía transrectal ofrece, ade­
más, la posibilidad de dirigir la biopsia hacia las zonas sospechosas.

La ecografía abdominal no tiene gran valor en la detección del car­


cinoma prostático. La TC y la RM tienen su principal papel en la es­
tadificación ganglionar y la valoración de metástasis a distancia. Las
primeras metástasis deben buscarse a nivel de los ganglios l infáticos de
las cadenas obturatrices e ilíacas.

Gammagrafía ósea

Se utiliza para la detección de metástasis óseas, tiene mayor sensibi­


lidad que la radiología convencional (Figura 20), y debe realizarse en
todo paciente en quien se sospeche metástasis (Gleason > 8, PSA > 20).
Antes de plantearse el tratamiento curativo, en ciertos pacientes con al­
tas probabil idades de encontrarse el cáncer extendido, se debe efectuar
una gammagrafía previa para confirmar la no existencia de metástasis
óseas o una TC para descartar metástasis ganglionares.

¡I) ETR corte transversal; {b} ETR corte longitudinal; (el Adenocarclnoma,
nódulo hlpoecojco en lóbulo derecho

Figura 19. Ecografra transrectal (ETR) de adenocarcinoma prostático Figura 20. Radiografía de columna. Metástasis osteoblásticas

23
Manual CTO de Medicina y Cirugía, 8," edición

D RECUERDA Agonistas LHRH. Aunque inicialmente ocasionan un aumento


de los niveles de testosterona, posteriormente suprimen la se�
las metástasis del cáncer de próstata son osteoblásticas, es decir, for­
man hueso (no en la gammagrafía, sino en la rad iografía) . creción de lH y de andrógenos. la elevación transitoria de los
andrógenos puede empeorar el cuadro clínico, principalmente
si existe compromiso medular por metástasis óseas. Esta eleva­
ción rflare-up) se debe suprimir mediante la administración de
Biop sia prostática antiandrógenos, previamente a la introducción de inhibidor de
la LHRH (MIR 00-0 1 , 1 1 9).
Debe realizarse para la confirmación del diagnóstico. Puede efectuarse Antiandrógenos (bicalutamida, flutamida, acetato de ciprotero.
vía transrectal o transperineal, guiada por el tacto rectal o bien guiada na). Compiten con el receptor androgénico. Suelen utilizarse con
por la ecografía transrectal, lo que añade efectividad a la prueba. la con agonistas de la lHRH. El acetato de ciproterona, además de
realización de la biopsia está indicada siempre que exista una anoma­ actuar como antiandrógeno, tiene un efecto progestágeno, por lo
lía del tacto rectal, elevación de los marcadores tumorales o alteración que actúa a nivel central, disminuyendo los pulsos de lH.
en las pruebas de imagen. la punción-aspiración con aguja fina (PAAF) Quimioterapia. No es muy efectiva en el adenocarcinoma pros­
es una alternativa con menores complicaciones, pero con el inconve­ tático. Se han realizado tratamientos con fármacos, que son una
niente de que no puede evaluar el grado histológico (Gleason). mezcla de un estrógeno y una mostaza n itrogenada (fosfato de
estramustina), pero los estudios son contradictorios y las res­

D RECUERDA puestas pobres. Existen estudios prometedores con el empleo de


docetaxel en pacientes con tumores hormonorresistentes.
Son indicaciones de b iops ia prostática el tacto rectal sospechoso, la
presencia de un nódulo ecográfico y un PSA > 4 (variable la cifra según
criterios).

Tratamiento por estadios (MIR 04-05, 1 06)

Estadio Tla. Tienen una mortalidad por la enfermedad del 2% a los


Trata miento 1 0 años, por lo que no precisan tratamiento, salvo quizás los pacien­
tes jóvenes (menores de 60 años) con una elevada esperanza de vida.
Estadio Tlb-Tlc. Alcanzan una mortalidad del 80% dejados a su
Opciones terapéuticas evolución natural. Por ello está indicada la prostatectomía radical,
la radioterapia externa o braquiterapia, en sujetos con esperanza de
Prostatectomía radical. los pacientes candidatos deben ser individuos vida superior a 1 0 años (MIR 02-03, 1 88) .
con una esperanza de vida superior a 1 0 años. Como complicaciones, Estadio T2a. Es la indicación más clara de prostatectomía radical. la
se puede encontrar incontinencia (2-57%), estenosis anastomótica radioterapia o braquiterapia se reservaría para pacientes de riesgo
(1 0%), impotencia (50%) o incluso la muerte « 5%). En l íneas gene­ quirúrgico elevado o que no aceptan efectos secundarios atribuibles
rales, suele ir acompañada de linfadenectomía íleo-obturatriz. a la cirugía.
Radioterapia. Como tratamiento curativo los resultados en estadios
I Estadio T2b y T2c. Un 40% demuestra ser en realidad estadio 3, tras
localizados se acercan a los de la cirugía. la diarrea crónica, la el análisis de la pieza qu irúrgica de prostatectomía radical (infraes­
proctitis, la cistitis rádica y las fístulas urinarias son complicaciones tadificación). la radioterapia externa o braquiterapia también puede
del tratamiento, así como la incontinencia y la impotencia a partir ser útil en pacientes de alto riesgo quirúrgico.
de los dos años de tratamiento. Estadio T3a. la i ndicación quirúrgica es dudosa, así como la radio­
Se ha empleado también radioterapia intersticial (braquiterapia) con terapia local, por lo que solamente se propondría a sujetos jóvenes,
implantación de yodo-1 23 (1- 1 23), oro-1 98 (Au-1 98), paladio e iri­ aun a costa de obtener malos resultados. Generalmente son tratados
dio. Su indicación queda limitada a tumores pequeños de estadio T1 como el grupo siguiente.
o T2, y sus resultados son similares a los de la cirugía. En caso de Estadio T3b, T4, N+, M+. Varón añoso con mal estado general. El
compresión medular o dolor por metástasis óseas, la radioterapia so­ tratamiento hormonal es la opción indicada. Puede ser preciso el
bre la metástasis puede conseguir el control local de la enfermedad. uso de radioterapia paliativa sobre la metástasis en caso de dolor
Hormonoterapia. El adenocarcinoma prostático está compuesto por (MIR 03-04, 9 1 ; MIR 98-99, 1 3 7).
una población heterogénea de células andrógeno-dependientes y
andrógeno-independientes. La supresión hormonal frena el creci­
miento de las primeras, pero no afecta a las a ndrógeno-indepen­ Recidiva tumoral posterior a tratamiento con intención curativa
dientes. Se puede conseguir disminuir los niveles de andrógenos
circulares por distintos métodos: Tras la realización de prostatectomía radical, los pacientes son monito­
Castración quirúrgica. Es el método aislado más eficiente, con la rizados generalmente con peticiones de PSA
ventaja de que elimina la necesidad de medicación permanente.
Por su rapidez en el efecto supresor hormonal, también está in­ Cuando las cifras de PSA tras prostatectomía radical son superiores a
dicada en las compresiones medulares por metástasis. 0,4 ng/ml, se considera recidiva bioquímica y debe hacer sospechar la
Estrógenos (dietilestilbestrol). Inhibe la secreción de LH. Actua­ existencia de metástasis a distancia, o bien la existencia de recidiva a
lemente este método se ha abandonado debido al alto riesgo nivel local (MIR 05-06, 1 06)_
cardiovascular que conlleva.
Progestágenos. Inhiben la secreción de lH y actúan como an­ Tras la realización de radioterapia como tratamiento de cáncer de
tiandrógenos, al unirse a los receptores de la dihidrotestosterona. próstata localizado, los descensos de PSA van siendo paulatinos (a di­
Es preciso añadir estrógenos para evitar el fenómeno de escape, ferencia de la prostatectomía radical) hasta conseguir un valor nadir,
que se produce tras varios meses de tratamiento. No son de uso que es el valor mínimo alcanzado tras el tratamiento y que se con­
habitual. siderará referencia para el seguimiento posterior. Existen diferentes

24
UrO,09¡a a
criterios para considerar el diagnóstico de recidiva bioquímica tras I ADENOCARCINOMA
"PB
tratamiento con radioterapia: cuando se constata la existencia de tres PROSTATICO

elevaciones sucesivas a partir del valor nadir, cuando se evidencian Locotluclón Zona translcional Periférica
n iveles nadir+2 (criterio de la ASCO), o con valores nadir+3 (criterio
Fases: 25% slndrome prostático
de Philadelphial. Compensación 25% retención aguda
Olnlca
CHnica 25% metástasis
Descompensaclón

Tratamiento de urgencia Tacto rectal


Ecografla transrectal (estadificación local)
Gammag rafla osea (metástasis óseas)
OIognOstlco
La compresión medular por el cáncer prostático no tratado puede ser la PSA (muy sensible, poco especifico). Descarta cáncer prostático,
d_.1
forma de presentación y constituye una urgencia importante. El objetivo pero no diagnostica HPB
Fosfatasa ácida (muy especifica, poco sensible)
del tratamiento debe ser la supresión androgénica rápida o la descom­ Biopsia (confirmación)
presión medular mediante laminectomía quirúrgica o radioterapia. Se
Fitoterapia Localizado: prostatectomla radical
pueden disminuir los niveles de andrógenos mediante castración quirúr­ Fármacos: finasterida, más Ilnfadenectomla bilateral.
gica urgente, ketoconazol en altas dosis o dietilestilbestrol i ntravenoso. Tratamiento n-bloqueantes radioterapia
Clrugla: adenomectomla: Avanzado: castración: quirúrgica
endoscóplca o abierta (elección), farmacológica
A continuación se expone una tabla que resume tanto las característi­
cas de la hiperplasia prostática benigna (HPB) como las del adenocar­ Tabla 1 1 . Tabla-resumen de las caracterfstlcas de la HPB
y del adenoca rcJnoma prostático
cinoma prostático (Tabla 1 1 l.

Casos clínicos representativos

Paciente de 66 años, intervenido de prostateclomía radical, hace 3 años por ade­ Un paciente de 67 años acude a Urgencias por presentar en los últimos días de­
nocarcinoma de próstata Cleason 8 (pT2b NOMO). Presenta, en el momento actual, bilidad progresiva de miembros inferiores, dificultad miccional e incontinencia
una cifra de PSA sérico de 1 2 nglml. Señale cuál de las siguientes afirmaciones le fecal. En la exploración física destaca cierta hipotonía anal, con una próstata muy
parece correcta: sugerente de malignidad al lacto, y debilidad de extremidades, conservando la sen­
sibilidad táctil. Con el probable diagnóstico de carcinoma de próstata melastásico,
1) La supervivencia media en el momento actual es menor de 1 año. ¿cuál de las siguientes opciones considera MENOS indicada para el tratamiento
2) La cifra de PSA está en rango normal ya que existen otras fuentes de producción de urgencia?
del mismo.
3) El paciente puede tener una recidiva local o bien melástasis a distancia. 1 ) Estrógenos intravenosos.
4) La utilización de bloqueo hormonal en este caso no es una opción de tratamiento 2) Análogos LHRH.
posible. 3) Ketoconazol (altas dosis) .
5) En caso de tratarse de una recidiva local, estaría indicado realizar cirugía de res.­ 4) Radioterapia.
cate para extirpar dicha masa. 5) Orquiectomía bilateral.

MIR 05-06, 106; RC: 3 MIR 00-01 , 1 1 9; RC: 2

Hombre de 77 años que refiere clínica de prostatismo de años de evolución, y que A la consulta acude un paciente de 54 años con molestias a la micción. Refiere
presenta elevación del PSA (antígeno prostático especifico) (89 ng/ml) y dolor en disminución del c,",orro, dificultad para el inicio, goteo postmiccional, sensación de
columna lumbar desde hace 2 meses. Al tado rectal, la próstata está aumentada de tenesmo y nicturia de tres veces. Presenta cultivos negativos y PSA de 2,1. En la
tamaño, de consistencia dura en ambos lóbulos, superficie nodular y límites mal defi­ ecograffa abdominal se objetiva una glándula prostática de 43 cc. Al tacto reclal
nidos. Tras realizarle una ecografía transrectal con biopsias prostáticas ecodirigidas, no se palpan nódulos sospechosos. En el IPSS obtiene una puntuación que permite
es diagnosticado de un adenocarcinoma de próstata pobremente diferenciado, que clasificar su sintomatología de moderada-grave. Su actitud deberá ser:
afecta a ambos lóbulos y que infiltra las vesículas seminales. La gammagrafía ósea
confirma la presencia de metáslasis en columna lumbar. ¿Qué tratamiento de los 1) Debido a su edad, el primer paso será iniciar tratamiento con Htoterapia.
siguientes aconsejaría en primer lugad 2) Debido a la gravedad de los síntomas se debe plantear cirugía de entrada.
3) Se debe iniciar tratamiento con a-bloqueantes.
1 ) Prostalectomía radical. 4) La mejor opción será i nicia r tratamiento combi nado con a-bloqueantes+inhibidores
2) Quimioterapia intensiva. de la 5 a-reductasa.
3) Hormonoterapia. 5) Se debe iniciar tratamiento con inhibidores de la 5 a-recluctasa.
4) Radioterapia pelviana externa.
5) Braquiterapia prostática. Re: 4

MIR 03-04, 9 1 ; RC: 3

25
06.
CARCINOMAS
DEL TRACTO URINARIO

Onentaclón Aspectos esenciales

MIR
Las preguntas sobre este
tema suelen ser senciHas
GJ El carcinoma vesical más frecuente es el urotelial, siendo el tabaco el principal factor de riesgo.
y repetitivas, aunque
últimamente ha aparecido (Il El carcinoma escamoso se relaciona con la esquistosomiasis (5. haemafobium).
como nuevo concepto el
carcinoma in silu. El estudio (I) El adenocarcinoma vesical se relaciona con el antecedente de exLrofia vesical.
del Desglose es especialmente
importante, pero se debe (i) El carcinoma papilar superficial y el carcinoma in situ (OS) son muy recurrentes.
tener en cuenta que la
tendencia parece orientada
a preguntarse cada vez más.
(I) Clínica más frecuente del carcinoma urotelial: hematuria, más típico con coágulos.
Es fundamental la parte de
tratamiento. m Cuando se trata de un carcinoma in situ: síntomas ¡rritativos (polaquiuria, disuria, tenesmo, etc).

m El mejor método para la estadificación local es la resección transuretral.

ill Prueba más sensible para el diagnóstico de OS: citología urinaria.

(2J Conducta ante un ClS: tratar con bacilo Calmette-Guerin (BCG ) y revisiones (cistoscopia y citologías).

G:Q) Actitud ante un tumor superficial: resección transuretral. Posteriormente, revisiones (CiSloscopia y citologías).

G:IJ Actuación ante un tumor infiltrante (afectación capa muscular): cistectomía.

6 . 1 . Carci n o m a vesical
E l carcinoma vesical es l a segunda neoplasia urológica e n frecuencia. Aparece más frecuentemente en varones
(2-3 : 1 ) y más en población blanca que negra. Su edad de máxima incidencia se sitúa entre los 60 y 70 años. De
el los, el 90% son carcinomas transicionales, el 8% escamosos y el resto adenocarcinomas. El epitelio uroteJial
recubre el tracto urinario desde las papilas caliciales hasta la uretra prostática, ambas incl usive. En cualqu iera
de estos niveles pueden desarrollarse los tumores uroteliales, correspondiendo la mayoría a la vejiga (más del
90%) y, más raramente, al tracto urinario superior (5%) o la uretra (1 %).

Entre los factores etiológicos (Tabla 1 2), se implican las ami nas aromáticas, presentes en las industrias textiles,
químicas y del caucho. El humo del tabaco es el principal factor de riesgo (50-60% aparecen en fumadores),
aumentando el riesgo a mayor consumo. También pueden jugar un papel importante los edulcorantes artificiales
(sacarina, ciclamato), la ciclofosfamida, los acetiladores lentos (mayor riesgo) y muchas otras posibles etiologías.
La infestación por Schistosoma haematobium aumenta la incidencia de carcinoma escamoso vesical, así como
la presencia de infección crónica o catéter vesical permanente.

Aminas arom�tlcas {2-naftllaminal; tabaco, industria


eL Preguntas textil, industria del caucho, colorantes
Fenacetinas crónicas Translclonales (90%)
- MIR 07-08, 102 Sacarina, ciclamato Mejor pronóstico
- MIR 06-07, 96 Ciclofosfamida (acrolelna)
- MIR 05-06, 103 Tabaco: ortofenoles, trlptófano
- MIR 04-05, 259
- MIR 03-04, 82 Schfsrosoma haematobium
Escamosos (8%)
- MIR 02-03, 1 79 litiasis, infecciones, catéteres
- MIR 00-01 , 1 1 8
Cistitis glandular
- MIR oo-O I F, 1 43, 144 Adenocarclnomas (2%)
Extrofla vesical
- MIR 98-99, 136
- M I R 98-99F, 1 4 5
Tabla 12. Factores etiológicos de los carcinomas del tracto urinario

26
UrOI09¡a a
El adenocarcinoma primario vesical es un tumor raro, aunque es el que
se ha visto asociado a la extrofia vesical con mayor frecuencia.

o RECUERDA
No hay que confundir Schisrosoma haemarobium con SchislOsoma
mansoni, que prod uce hipertensión portal.

Histología e historia natural

Haciendo referencia al carcinoma de células transicionales, hay que di­


ferenciar tres formas de la enfermedad con comportamiento, pronóstico
y tratamiento completamente distinto (Tabla 1 3 y Figura 2 1 ). El 70% de
los tumores vesicales se presentan como tumores papilares de crecimien­
to principalmente endocavitario y frente de invasión único. Un 1 0% son
1. Mucosa ----;.� Ta,Tis 4. Grasa -----+J T3
sólidos, con invasión tentacular en profundidad y extensión linfática y vas­
2.5ubmucosa J T1 5. Órganos vecinos _ T4
cular temprana. El 20% restante son formas mixtas. El primer grupo suele
3. Muscular ----+J 12
corresponder a tumores superficiales de bajo grado histológico, mientras
que los sólidos, con mayor frecuencia, son tumores infiltrantes de grado
histológico más elevado. la principal característica de los tumores papi­ Figura 2 1 . Esquema de la estadificación del tumor vesical
lares superficiales es la recurrencia, que ocurre en un 50-75%, según el
grado y estadio. El 25% recurrirán y progresarán en grado y estadio, y úni­
camente e1 1 5% acabará desarrollando un tumor infiltrante o metastásico. Se puede encontrar en el ámbito vesical disti ntas lesiones benignas que
no se asocian con el desarrollo de cáncer: los nidos de von Brunn, la
la mayoría de los tumores infi ltrantes se encuentran confinados a la cistitis quística y glandular origi nados en procesos inflamatorios o ¡rrila­
vejiga en el momento del d iagnóstico, y sólo un 20-25% presentan tivos cróni cos, y que probablemente sean distintas manifestaciones de
extensión ganglionar o metastásica. El 50% desarrollarán metástasis a un mismo proceso, aunque pueden plantear el diagnóstico diferencial;
distancia, a pesar del tratamiento (MIR 04-05, 259). (ocasionalmente se han descrito adenocarcinomas vesicales asociados
a la cistitis glandular). Olras lesiones benignas serían el adenoma nefro­
El tercer grupo que merece mención aparle es el carcinoma in situ. A génico, el pólipo simple, el papiloma i nvertido o el papiloma velloso.
pesar de encontrarse l imitado al urotelio, por lo que es superficial, está
formado por células poco diferenciadas con displasia grave. Tiene una
alta tasa de recidiva y progresa hacia tumor i nfiltrante en el 50-75% de Diag nóstico
los casos. Este mal pronóstico le confiere un carácter completamente
distinto del carcinoma in siw de otras regiones, en las que se considera
el estadio inicial de la enfermedad tumoral. El carcinoma in situ puede la hematuria macroscópica o m icroscópica monosintomática es el ha­
estar asociado a focos de carcinoma superficial (26%) o ¡nfiltrante (60%) llazgo más frecuente, presente en el 75% de los pacientes (MIR 06-07,
o bien encontrarse de forma aislada, siendo generalmente multifocal 96). la presencia de microhematuria asintomática, descubierta durante
tanto en vejiga como en otras puntos del uratelio (MIR 07-08, 1 02). estudios de cribado, sólo se relaciona con enfermedad significativa en
menos del 2% de los casos. Pueden encontrarse síntomas irritativos
(escozor, polaquiuria, tenesmo) en el 25-30%, solos o acompañando a
la hematuria. la presencia de un síndrome cistítico no justificado por
i nfección o litiasis debe hacer sospechar la presencia de un carcinoma
Tis: carcinoma in situ (plano) vesical, especialmente por su asociación con el carcinoma in situ (MIR
Ta: carcinoma papilar no infiltrante 00-01 F, 1 44). Con menor frecuencia, el paciente consulta por dolor en
T1: tumor que invade tejido conjuntivo subepitelial
T2: tumor que invade músculo
flanco por obstrucción ureteral, pélvico o por edema en miembros in­
- T2a: tumor que invade la mitad interna feriores (extensión l infática). la exploración física suele ser i rrelevante,
- T2b: tumor que invade la mitad externa salvo en la enfermedad avanzada.
T3: tumor que invade tejido perivesical:
• T3a: microscópicamente

- T3b: macroscópicamente (masa extravesical) las citologías urinarias son una prueba sencilla y fiable que debe ser
T4a: tumor que invade próstata, útero o vagina rea l izada en todos los casos de hematuria asi ntomática o sospecha de
T4b: tumor que invade pared pélvica o pared abdominal
tumor vesical. Su sensibilidad depende del grado de diferenciación
del tumor vesical, alcanzando el 75-1 00% en tumores de alto grado
y carcinoma in situ, siendo en este último un método diagnóstico más
Nx: metástasis ganglionar regional desconocida
NO: ausencia de metástasis ganglionar regional rentable que la ecografía, la Te, la urografía o la biopsia múltiple. Son
N1: metástasis a un solo ganglio entre 2 y 5 cm especialmente útiles en el seguimiento de pacientes sometidos a resec­
N2: metástasis en un ganglio mayor de 5 cm o múltiples no mayores de 5 cm
ción transuretral en combinación con la cistoscopia.
N3: metástasis mayores de 5 cm

Entre las pruebas radiológicas destacan la ecografía (con una sensibi l i­


MO: no metástasis dad del 80%, pera poco útil para el d iagnóstico de las neoplasias del
M l : metástasis a distancia tracto urinario superior (Figura 22]) y la urografía intravenosa (capaz de

Tabla 13. Estadificación del carcinoma vesical


detectar la presencia de tumor en el 60% de los casos).

27
Manual eTO de Medicina y Cirugía, B.a edición

un 40%, siendo también tratamiento de elección en el carcinoma


in situ, con una eficacia del 70% (MIR 05-06, 1 03; M I R 02-03, 1 79;
MIR 00-0 1 , 1 1 8). Sin embargo, no se utiliza en los tumores vesicales
como primera línea, reservándose para los tumores de riesgo o con
múltiples recidivas, ya que también se acompaña de mayor número
de efectos secundarios y de complicaciones, entre los que se en­
cuentran cistitis febril, síndrome pseudogripal, y las más graves de
sepsis, prostatitis granulomatosa, neumonitis e incluso muerte. Estos
cuadros precisan tratamiento tuberculostático completo al menos de
tres a seis meses.

En la enfermedad i nvasora O infiltrante, tras la resección transure­


tral para evaluar la afectación parietal, el tratamiento de elección
es la cistectomía radical. La quim ioterapia sistémica, adyuvante o
neoadyuvante, no parece mejorar los resultados de la cirugía aislada.
Igualmente se puede decir de la radioterapia, que únicamente ha de­
mostrado i ncrementar el tiempo libre de recidiva local, s i n aumentar
la supervivencia.

La quimioterapia combinada con M-VAC (metotrexato, vinblastina,


adriamicina, cisplatino) se reserva para pacientes en los que existe
En la urografía i ntravenosa, además de la presencia de defectos de afectación ganglionar o metastásica, con respuestas completas en el
repleción, se puede sugerir el diagnóstico de tumor vesical, rigidez 1 5-30% y parciales en el 30-40%. Tras la cistectomía, los uréteres son
y falta de distensibilidad vesical, la obstrucción de un uréter o el derivados generalmente a segmentos intestinales o a piel, pudiendo
desplazamiento de la vejiga, entre otros. En caso de dudas sobre el realizarse estomas no continentes o reservarías continentes directa­
tracto urinario superior, se recurrirá a la pielografía retrógrada en el mente a l remanente uretral.
momento de realizar la cistoscopia. Ésta es fundamental para la eva­
luación del tumor vesical; puede real izarse bajo anestesia local cuan­
do existan dudas con las pruebas realizadas previamente, pero si el
o RECUERDA
la adriamicina (doxorrubicina) es un quimioterápico que puede produ­
diagnóstico de presunción es firme, y dado que en todo tumor debe cir cardiotoxicidad.
realizarse resección transuretral para evaluar el grado de infi ltración,
se puede esperar a tener al paciente en quirófano bajo anestesia gene­
ral o raquídea para practicarla (MIR 00-01 F, 1 43). En un 1 0% de los
casos de cistoscopia con citologías positivas no se encuentra tumor
en la vejiga, lo cual puede ser debido a la presencia de carcinoma in 6.2. Tu mores del tracto u r i n a rio
situ, tumor en vías urinarias altas, a un carcinoma ductal de próstata
o a un falso positivo de la prueba (generalmente por inflamación de s u perior
la pared vesical o por tratamiento concomitante con radioterapia o
quimioterapia endovesical).
Entre e l 2-5% d e los tumores uroteliales se encuentran localizados
La Te se utiliza para la estadificación de la enfermedad infi ltrante, ya entre los cál i ces y los orificios ureterales. En su etiología están
que aporta información limitada sobre la infiltración tumoral. La RM i m p l i cados los mismos factores que para el carcinoma vesical, a
consigue mejores imágenes de la cúpula vesical por sus cortes sagita­ los que habría que añadir la nefropatía por abuso de analgésicos
les, pero no aporta mayor información que la TC (fenacetinas) y la nefropatía de los Balcanes. En el 70-80% de los
pacientes aparece hematuria macroscópica, siendo el dolor có­
lico por obstrucción ureteral la segunda queja en frecuencia de
Tratamiento aparición.

Todo el proceso diagnóstico va encaminado a establecer si el tumor


o RECUERDA
la hematuria también es la manifestación más frecuente del hiperne­
vesical es superficial o infiltrante, ya que el tratamiento varía radical­ froma.
mente en función de este hecho.

Los tumores superficiales son manejados mediante resección transu­


retral . Dada la alta frecuencia de recidivas, la mayoría se tratan poste­ Diag nóstico
riormente con i nstilaciones endovesicales (quimioterapia o inmunote­
rapia local) que disminuyan la aparición de nuevos tumores. Entre los
quimioterápicos empleados se encuentran la mitomicina, la tiotepa, la La sistemática d iagnóstica es básicamente la misma que para el
adriamicina o la epirrubicina. Con todos ellos se consigue reducir las tumor vesical. Es decir, ecografía (detectará hidronefrosis del lado
recidivas alrededor de un 20%. del tumor), UIV (se verá un defecto de repleción no compatible
con u n cálculo o una anulación funcional de ese sistema excretor),
La inmunoterapia endovesical con BCG (bacilo Calmette-Guerin) es, y Te para la estadificación (Figura 23) ( M I R 98-99, 1 36; M I R 98-
sin duda, la más eficaz, disminuyendo el porcentaje de recidivas en 99F, 1 45 ) .

28
UrOIOgía �
ureteropieloscopia, elemento que, además de diagnóstico, puede
tener un fin terapéutico en tumores de pequeño tamaño y aspecto
superficial (Figura 241.

Fumador + Hematuria

Sospecha d. tumor uroteUal

+
Citologias + Pruebas de imagen
<: ECO 'VP
UIV

t t
No concluyente Concluyente
para tumor vesical para tumor vesical

t t
Cistoscopia ES -------- ==. RTUV
� '-=':= -;:
:: = :...�.' �� :::;�--l
6t t t t
Biopsia vesical ES ----Tis
--�.� 2:12 TA,Tl

6t t

� ¿ �
Sospechar tumor BCG Cistectomfa Quimio BCG (G3)
Tracto urinario superior
Revisiones
· Ureterorrenoscopia
Cistoscopia
· Pielografía retrógada ReCi iva TíS ClOmia R diva _
+ Citología
· Citologías selectivas
· Cepillado ureteral

Ftgura 24. Algoritmo diagoóstico-terapéutko en tumor de vejiga (MIR 03-04,82)

Tratam iento

El tratamiento estándar es la nefroureterectomía radical con extirpación


de un rodete perimeático vesical; tal extensión es necesaria por la ten­
dencia a la recidiva de estos tumores. Es planteable, sin embargo, el
empleo de tratamientos conservadores en caso de tumores papilares,
Figura 23. Ecografia.Tumor vesical con zonas calcificadas no infiltrantes, únicos y de pequeño tamaño.

En estos tumores, la citología urinaria aumenta su eficacia si se ob­ Es necesario un seguimiento posterior de la vejiga y el riñón contra la­
tiene de forma selectiva, cateterizando el uréter del lado afectado. teral, ya que un 20-30% desarrollarán un tumor vesical metacrónico y
Otros elementos de diagnóstico son las biopsias por cepillado y l a un 2% en el sistema colector contralateral .

Casos clínicos representativos

Varón de 5 7 años de edad, fumador, que consulta por hematuria terminal, polaquiu­ Mujer de 63 años que es diagnosticada de carcinoma de células escamosas del Irí­
ria, urgencia y dolor miccional. Presenta citologías urinarias positivas de carcinoma gono vesical, con invasión de la capa muscular. ¡Cuál seria su actitud lerapéutica
urotelial, y el estudio anatomopatológico tras la resección transurelral es de carci· en este caso?
noma in silu difuso, con intensa inflamación crónica. El tratamiento estándar será:
1) Radioterapia externa con 7.000 rads.
1 ) Instalación del bacilo de Calmette-Guerin. 2) Quimioterapia adyuvante, seguida de cistectomía radical.
2) Cistectomía radical. 3) Resección transuretral, seguida de inmunolerapia intravesical (BCG).
3) Instilaciones con mitomicina. 4) Cistectomía radical con extirpación de cara anterior de vagina.
4) Quimioterapia con cisplalino. 5) Radioterapia externa, seguida de quimioterapia con cisplatino.
5) Antiinllamatorios no esteroideos más quinolonas durante seis meses.
MIR 03-04, 82; RC: 4
MIR 05-06, 1 03; RC: 1

29
Manual CTO de Medicina y Ciru g ía, 8.... edición

Casos clínicos representativos

Un paciente de 64 años acude a consulta por síndrome miccional de dos meses de Una paciente de 58 años acude por polaquiuria de dos años de evolución, nicturia
evolución. Refiere que sU MAP le ha dado tratamiento antibiótico y le ha realizado y dolor hipogástrico. Niega haber visto sangre en la orina y es fumadora. Señale la
cultivos que han sido negativos. Se le realiza una ecografía y una flujometría que son respuesta correcta:
normales y una urografía intravenosa que no evidencia alteraciones. En la cistosco­
pia no se observan lesiones intravesicales. las citologías urinarias son sugestivas de 1) Se deben realizar biopsias a esta paciente.
malignidad. Señale la incorrecta� 2) Una dstoscopia permitirá confirmar el diagnóstico.
3) los tratamientos empleados hasta el momento han demostrado una gran eficacia.
1) El tratamiento de elección será la inmunomodulación vesical. 4) Muy probablemente los cultivos serán positivos.
2) Es imprescindible la realización de biopsias para confirmación del diagnóstico. 5) En todos los casos se detectarán > 1 0 hematíes/campo en el sedimento de orina.
3) Si recidiva tras las instilaciones, se deben repetir los ciclos dos veces más.
4) la cistectomía es el tratamiento final en muchos de estos pacientes. Re: 1
5) A priori no se encontrará infiltración de la capa muscular.

RC: 3

30
07.
TUMORES TESTICULARES

Ont>ntil(IÓn Aspectos esenciales

MIR
Estc tema es sencillo m El tumor testicular es la neoplasia sólida más frecuente en el varón joven.
y rentable siempre
que se seleccione o la tasa de curación es mayor del 90%.
lo realmente importante:
El tumor testicular más frecuente es el seminoma. No obstante, ésto es muy variable según la edad del pa·
la clínica y el diagnóstico. [I)
Es fundamental la revisión ciente.
profunda de los Desgloses.
En cuanto al tratamiento, dado Una masa testicular por encima de los 50 años debe hacer pens.lr en un linfoma.
que no existe un protocolo
universalmente aceptado, El tumor típico de las disgenesias gonadales es el gonadoblasloma.
es mejor conocer ideas
generales. la clínica más frecuente es una masa escrotal indolora.

El seminoma nunca produce a·fetoproteína .

Como tratamiento, la orquiectomía por vía inguinal se realiza en todos los casos.

El seminoma es radiosensible. Por ello puede utilizarse radioterapia como tratamiento en los primeros esta­
dios. Si se trala de un estadio avanzado, se emplearía quimioterapia.
los lumores no seminomalosos se tratan con orquieclomía + quimioterapia. No obstante, si es un tumor
limitado al testículo, puede plantearse la vigilancia tras la orquiectomía.

7 . 1 . Eti ología y epidemiolog ía


Los tumores testiculares suponen el ' ·2% de las neoplasias en varones, y son las neoplasias sólidas más frecuen­
tes entre los 20 y 35 años. Presentan mayor incidencia a mayor desarrollo de la sociedad, y también en la raza
blanca más que en las demás. La tasa de curación es superior al 90%.

El 95% de ellos proceden de células germinales y, aunque globalmente el semi noma es el más frecuente, la in·
cidencia varía según el grupo de edad considerado. El 5% restante se reparte entre tumores del estroma gonadal
(1 ·2%), ! infamas (1 %), gonadoblastomas (células germinales y del estroma) metástasis y otros.

Los factores de riesgo para el desarrollo de tumor testicular son: teste h i poatrófico, síndrome de Klinefelter,
h i storia fam i l iar de neoplasias testiculares, tumor previo en el otro teste y criptorquidia. Los testículos no
descendidos tienen mayor riesgo de desarrollar tumores, y esta probabilidad aumenta si la situación del
teste es intraabdominal. Asimismo, el testículo contra lateral, aunque de localización escrotal, tiene una
incidencia mayor; de hecho, el 20% de los tumores en pacientes con criptorquidia se desarrollan en el
testículo no criptorquídico. Por estas razones, los testículos no descendidos deben descenderse, preferi·
blemente entre el primer y segundo año de edad para facilitar su seguimiento y, con ello, la detección de
1 Preguntas un eventual tumor.

- MIR 09- 1 0. 102 Una vez alcanzada la pubertad, un teste criptorquídico no descendido probablemente deba extirparse, dado que
- MIR 06-07, 94
estos testículos pierden la capacidad de espermatogénesis y conservan su potencial malignizante. No obstante,
- MIR 05-06. 107
- MIR 04-05, 107 hay autores que prefieren no extirparlos, siempre y cuando se puedan descender a la bolsa escrotal, ya que aún
- MIR 03-04, 145 pueden mantener su función endocrina (secretora de testosterona) (MIR 99·00, 1 86).
- MIR 02-03, 186
- MIR 01 -02, 108
- MIR 00-01 F, 142 Otros factores que se pueden encontrar relacionados son las hernias inguinales infantiles (no demostrado) y
- MIR 99-00, 186
- MIR 99-00F, 145
la orquitis urliana secundaria al paramixovirus causante de la parotiditis (siempre que haya producido atrofia)
- MIR 98-99F, 2 1 8 y, entre los factores tóxicos, la exposición a radiaciones, fuentes de calor, productos para teñido del cuero y

31
Manual eTO de Medicina y Cirugía, 8.3 edición

estrógenos ¡ntra útero durante el primer trimestre del embarazo (MIR saco vitelina en el 38% de los tumores testiculares del adulto.
98-99F, 2 1 8). Aunque entre un 8 y 25% de los pacientes presentan Coriocarcinorna. En el estudio histológico tiene que contener ele­
historia de traumatismo testicular, todos los autores parecen estar de mentos de sincitiotrofoblasto y citotrofoblasto para ser considerado
acuerdo en que éste supone más bien el motivo por el que se descubre como tal. En el momento del diagnóstico, generalmente existen me­
una masa escrotal, y no su origen. tástasis a distancia (suelen ser vía hematógena) y un tumor primario
testicular pequeño. Raramente es puro (MIR 06-07, 94).
Teratoma. Por definición, se encuentra formado al menos por dos capas
distintas de células germinales (endodermo, mesodermo o ectodermo).
7.2. Anatomía patológica Teratocarcinorna. Tumor mixto con áreas de teratoma y de carcino­
ma embrionario. Un 64% tiene también áreas de semi noma.
Conadoblastoma. Contiene grandes células similares al seminoma
Los tumores testiculares se suelen originar como una masa testicular y otras menores, como células de Sertoli inmaduras o de la granulo­
intraparenquimatosa. A partir de ahí, pueden quedarse localizados o sa. Se asocia preferentemente a las gónadas disgenéticas y estados
metastatizar. La localización más frecuente de metástasis son los gan­ intersexuales.
glios retroperitoneales (casi siempre es el primer escalón en la disemi­ Tumores de células de Sertoli. Forman áreas tubulares similares a
nación)¡ posteriormente puede aparecer la afección mediastínica, de los túbulos del testículo prepuberal normal. De comportamiento be­
vísceras abdominales y de pulmón. nigno, es excepcional que metastaticen.

Hay que tener en cuenta que el testículo izquierdo drena a los ganglios
paraaórticos y preaórticos a nivel de L2. El derecho drena a ganglios
• RECUERDA
El lumor de células de Serlol i se ha asociado al síndrome de Peutz­
interaortocavos, precavas y preaórticos, y también a nivel del hilio re­ Jeghers (véase Apa rtado de Síndromes de poliposis, en la Sección de
nal. La diseminación hematógena es menos frecuente, salvo en el co­ Digestivo y cirugía general).
riocarcinoma, vía vasos espermáticos, siendo los puntos más habituales
de metástasis: pulmón, hígado, hueso y SNC, por orden de frecuencia.
Tumores de células de Leydig. Pueden verse cristaloides en su cito­
Los tumores testiculares se dividen en dos grupos: tumores que no deri­ plasma. Generalmente son benignos. Pueden ser hormonal mente acti­
van de las células germinales (5%) y tumores que derivan de las células vos, produciendo pubertad precoz o feminización (MIR 99-00F, 145).
germinales (95%); a su vez, estos últimos se dividen en seminomatosos Linforna. Tanto como localización secundaria o como origen pri­
y no seminomatosos (Tabla 1 4). mario (menos frecuente), una masa testicular en un varón mayor de
50 años sugiere en primer lugar el diagnóstico de linfoma. Tras la
TUMORES DE CHULAS GERMINALES TUMoRES DEL ESTRoMA orquiectomía o la biopsia testicular para el diagnóstico de certeza,
Tumonos de un tipo
el tratamiento no varía respecto a los l infomas habituales.

$eminoma
- Típico
- Anaplásico
- Espermatocítico 7.3. C l í n i ca
Carcinoma embrionario
PoHembrioma
Tumor del �co vitelina (seno endodérmico)
Coriocarcinoma · Tumores células Leydig La manifestación más frecuente es como masa escrotal indolora. Con
Tumores células SertoH
Teratoma: mucha menor frecuencia, el motivo de consulta está originado por la
·

Tumores estructuras
- Maduro ·

gonadales primitivas presencia de metástasis ganglionares (masas supraclaviculares o abdo­


- I nmaduro
- Con transformación maligna minales (MIR 09-10, 1 02), o efectos endocrinos (ginecomastia, puber­
tad precoz).
Tumores de m6s de un tipo hIstcJI6gIco

Teratocarcinoma
Otros
EI 1 0 % de los tumores se presentan como escroto agudo en la urgencia.

Tumores germinales + estroma'"

Gonadoblastoma • RECUERDA
Exislen algunas enfermedades, como la sarcoidosis, que aumentan el
Tabla 14. Tumores de testkulo. Clasificación histológica
ta maño testicular sin existir un tumor.

Serninorna. Puede aumentar el tamaño testicular hasta 1 0 veces sin


distorsionar su morfología. Ocasionalmente se presenta extrates­
t¡cular en mediastino (la más frecuente), retroperitoneo, o región
pineal de forma primaria. Se han descrito tres tipos histológicos: 7.4. Diag nóstico
seminoma típico, anaplásico y espermatocítico (más frecuente en
individuos mayores de 50 años). El seminoma puro no es productor
de marcadores tumorales, pero hasta en un 1 5% de los casos pue­ La exploración física de una masa indolora, sin signos inflamatorios y
den aparecer células del sincitiotrofoblasto, produciendo elevacio­ de largo periodo de evolución, debe hacer sospechar un tumor testicu­
nes de la p-HCG. lar (Tabla 1 5).
Carcinoma embrionario. Tiende a metastatizar de forma temprana.
Forma masas más irregulares y heterogéneas que otros tumores. Un elemento i mportante, tanto para su diagnóstico como para el segui­
Tumor del seno endodérmico. Este tumor fue descrito inicialmente miento, son los marcadores tumorales: a-fetoproteína (AFP) y fracción
como una forma rara de tumor infantil. Se encuentran elementos de p de la gonadotropina coriónica humana (P-HCG) (Tabla 1 6).

32
UrOIOgía �
presencia de enfermedad residual, ya que la elevación persistente de
I ntratubular uno de estos marcadores después del tratamiento supone la existencia
pTls
pTl Testículo y epididimo sin invasión vascular/linfática de tumor no eliminado.
pT2 Testículo y epidídimo con invasión vascular/linfática o túnica
vaginal La ecografía testicular es un método sencillo y fiable para la diferen­
pTl Afectación de cordón espermático
ciación entre masas sólidas y quísticas, y su localización exacta i ntra­
pT4 Escroto
testicular o dependiente de los anejos. Cuando, a pesar de los marca­
N1 Ganglios menores de 2 cm
dores, los datos ecográficos son sugerentes de tumor, está indicada la
N2 Ganglios entre 2-5 cm
N3 Ganglios mayores de 5 cm exploración quirúrgica a través de una incisión inguinal, para evitar la
posibi lidad teórica de implantes tumorales en la piel escrotal y tener
M1. Metástasis en ganglios no regionales o pulmón
M1b Metástasis viscerales no pulmonares mejor control de pedículo vasculolinfático a nivel del cordón espermá­
tico (MIR 01 -02, 1 08).
Sx Marcadores tumorales séricos no disponibles
SO Niveles de marcadores normales
51 lDH < 1.5 x n ; y p-HCG < 5.000; Y AFP < 1 .000 Si la exploración confirma la presencia de una masa, el testículo debe
52 lDH entre 1,5 x n y 10 x n o p-HCG entre 5.000 y 50.000 ser extirpado (orquiectomfa radical).
o AFP entre 1 .000 Y 10.000
53 LDH > 10 x n o j3-HCG >50.000 o AFP > 1 0.000
La evaluación de la extensión tumoral se completará mediante TC to­
racoabdominal. Así se constatará si la enfermedad está limitada al tes­
tículo (estadio 1), o existe afectación de ganglios infradiafragmáticos
(estadio 11), o bien si hay incluso extensión supradiafragmática o a ór­
ganos sólidos (estadio 1 1 1). El sistema de estadificación utiliza múltiples
variaciones, pero quizá la clasificación más aceptada sea la expuesta
anteriormente en la Tabla 1 5 .

7 . 5 . Diagnóstico d ife rencial


Tabla 15. Estadificaci6n clínica y patológica

Aunque frecuentemente l a existencia d e u n tumor testicular n o plan­


Tumor con una palabra aumenta ji-HCG tea dudas diagnósticas, existen una serie de patologías testiculares
Coriocarcinoma que, junto con los tumores, pueden presentarse en algún momen­
- Seminoma (en la mujer, el equivalente es el disgerminoma)
to dentro del cuadro genérico que se denomina "escroto agudo",
Tumor con dos palabras aumenta a.-fetoproteina caracterizado por el aumento doloroso de volumen del contenido
- Carcinoma embrionario escrotal, acompañado o no de signos inflamatorios (Tabla 1 7 y Fi­
- Tumor del seno endodérmico
gura 25).
Tabla 16. Marcadores tumorales en tumores germinales Orquiepididimitis: suelen presentarse con dolor i ntenso, enro­
(válidos para ovario y testiculo) jecimiento cutáneo, fiebre, y a veces si ntomatología miccional.
En la exploración, la elevación del testículo (signo de Prehn)
La AFP es sintetizada por células del saco vitelino y, por tanto, está disminuye el dolor (signo de Prehn positivo). En su etiología se
presente en tumores de saco vitelino o seno endodermal, y en los deben considerar gérmenes de transmisión sexual en pacientes
carcinomas embrionarios. El seminoma nunca produce AFP (MIR adultos menores de 35 años, y uropalógenos (E. coli) si superan
04-05, 1 07; M I R 02-03, 1 86). En cualquier caso, se debe tener en esla edad.
cuenta que la AFP es u n marcador i nespecífico, y se podría en­ Torsión del cordón espermático: suele aparecer en la infancia o
contrar elevado en enfermedades hepáticas benignas o malignas, la adolescencia, con dolor de aparición brusca y signos cutáneos
algunas neoplasias pancreáticas y de la vía biliar o en la ataxia­ inflamatorios crecientes a medida que progresa el cuadro. El teste
telangiectasia. se encuentra horizontal izado, y ocasionalmente puede palparse la
espiral del cordón torsionado. En este caso, la elevación del testícu­

a RlECUpERDAb· · ' . . '


lo i ncrementa la sensación dolorosa.
Hidrocele y espermatocele: son dos cuadros que raramente se
a AF tam len se e Ieva en l IqUl-do amOlotlCO
. •
en a IteraClones dei CIerre
del tubo neural. Ver Ginecología. presentan de forma brusca y con dolor agudo, fácil mente diferen­
ciables por la exploración y su transiluminación positiva y, ante la
duda, mediante ecografía.
La fracción p de la HCG es producida por las células del sincitiotrofo­
blasto presentes en el coriocarcinoma, y también de forma ocasional y
Polaridad conservada
de forma aislada, en algunos semi nomas (MIR 00-01 F, 1 42). Orquitis
Prehn MEJORA el dolor

En conjunto, el 70% de los tumores testiculares producen algún marca­ Testículo horizontalizado
Torsión del cordón espermático
dor, luego existe hasta un 30% de tumores con marcadores negativos Prehn EMPEORA el dolor

al diagnóstico.
Tumor testicular Masa palpable indolora
La vida media de la a-fetoproteína es de siete días, frente a tres días
Tabla 17. Diagnóstico diferencial de los tumores testiculares
de la �-HCG. Este dato es importante a la hora de valorar la posible

33
Manual eTO de Medicina y Cirugía, 8.a edición

D RECUERDA
El cisplatino produce vómitos con mucha frecuencia. Otro efecto secun�
dario es su nefrotoxicidad.

Estadio lIa-lIb. En este caso, el tumor ya está extendido a gangl ios


retroperitoneales, y requiere, por tanto, tratamiento agresivo. Se
dispone de dos opciones: radioterapia sobre las cadenas afectas
(teniendo en cuenta el teste afectado, se irradiará a unas cadenas
ganglionares u otras), o quimioterapia BEP (cisplatino, etopósido y
bleomicina). Ambos obtienen resultados muy similares
Estadio 11e-1II (estadios avanzados). El tumor tiene metástasis gan�
glionares retroperitoneales superiores a 5 cm, O afectación supra�
diafragmática o de vísceras sólidas. La radioterapia deja de ser una
opción terapéutica. La quimioterapia (BEP) es la única posibilidad,
presentando una tasa de curación de alrededor del 80%.

Tu mores no seminomatosos

Estadio 1. Existen tres posibilidades terapéuticas tras la orquiectomía:


Observación y seguimiento periódico. Tasas de recaídas de un 20%.
linfadenectomía retroperitoneal de estadificación, pues así se
tiene certeza del estadio y se reduce a un 1 0% las recidivas (en
Europa no está extendida esta práctica).
Quimioterapia profiláctica (cisplatino), y así se reducen al 5%
Figura 25. Pieza de tumor testicula r las recidivas.
En aquellos casos en que exista invasión vascular en la pieza de
orquiectomía, la tasa de recidivas asciende al 50%. Parece lógico,
en estos casos, inclinarse de entrada por una de las dos últimas op�
7.6. Trata m i e nto ciones. La tasa de curación alcanza el 98%.
Estadio lIa-lIb. Históricamente se realizaba linfadenectomía retro�
peritoneal completa como único tratamiento, pero ante tasas de
Hasta la fecha no existe un protocolo único de tratamiento, pudiendo recidiva no desdeñables, actual mente se indica quimioterapia de
variar incluso de un centro a otro. Siempre se realizará orquiectomía inicio (BEP). La tasa de supervivencia supera el 95%.
radical vía inguinal. Posteriormente, el patólogo informará de la estir­ Estadio 11e-1II. Antes de la aparición de la actual quimioterapia, la
pe histopatológica. A continuación, el estudio de extensión mediante supervivencia era del 5·1 0%. Actualmente, el esquema de trata­
Te toracoabdominopélvica y nuevos marcadores postorqu iectomía. En miento mayoritariamente aceptado es quimioterapia primaria.
función de la histología y del estadio, se asignará un tratamiento a cada
enfermo. El tratamiento de la neoplasia testicular se resume en la Figura 26.

D RECUERDA TUMOR TESTICULAR


La orquiectomía en el cáncer de testículo es v ía inguinal. En el cáncer
de próstata , vía escrotal.
t
Orquiectomía radical

NO SEMINOMA
Seminoma SEMINOMA

t t t t
la tla,lIb
lIa,lIb

Se caracteriza por su gran radiosensibilidad, de ahí que la radioterapia • Observación


t Observación +++ t
Si FR : Linfadenectomra invasión Quimioterapia
haya sido la base del tratam iento de estos tumores. Actualmente, l a •
Radioterapia
> 4 cm rere testis Quimioterapia
(EEUUI vascular
quimioterapia obtiene resultados similares. Quimioterapia o
Estadio l. El tumor teóricamente está l imitado al testículo. No obs� (UEI linfática
Radioterapia
tante, se sabe que hasta un 20% de pacientes presentan micro�
Quimioterapia
metástasis a ganglios retroperitoneales, que en el momento del
diagnóstico no se detectan. La presencia de micrometástasis se ha SEMINOMA I NO SElIllfIOMA
relacionado con dos factores de riesgo: tumor de más de 4 cm y la
invasión tumoral de la rete lestis. En estos enfermos se aconseja ad�
ministrar radioterapia o quimioterapia (cisplatino). Con ésto, frente Quimioterapia
a los pacientes que optaron únicamente por observación, la recidiva
desciende del 20 al 5%. Figura 26. Algoritmo de tratamiento de la neoplasia testicular

34
urOlogja �
Masas residuales

Se define como masa residual la existencia de conglomerados adeno­ MASA RESIDUAL

páticos tras tratamiento quimioterápico o radioterápico. Cuando el tu­


mor primario es un semi noma y existen masas residuales, la actuación
a segui r será: si la masa residual es inferior a 3 cm, tiene muy pocas SEMINOMA NO SEMINOMA

probabi lidades de contener tumor residual y no requiere más que ob­


servación. Pero cuando es superior a 3 cm, se debe realizar una PET < 3 cm > 3 cm
(tomografía por emisión de positrones), si existiera esta posibilidad a
+
nivel técnico, pues detecta con una alta sensibilidad y especificidad la
presencia de tumor residual. Si no se dispone de una PET o si ésta es
positiva, se realizará cirugía de la masa.
1
Observación -+---
+/- PET

e ;/ "'- $

Cuando el tumor primario es un tumor no seminomatoso, se debe rea­


¡
Exéresis de la masa residual
lizar exéresis de dicha masa siempre, con independencia del tamaño.

La histología de estas masas residuales, una vez extirpadas, puede ser: 50% necrosis 35% teratoma 15% tumor viable

tejido necrótico hasta en un 50% de las ocasiones, tumor viable en un


1 5% y teratoma en un 35% (éstos, dejados a libre evolución, pueden
convertirse en teratomas malignos o producir procesos compresivos
Figura 27. Algoritmo de tratamiento de las masas residuales
con su crecimiento) (Figura 27).

Casos clínicos representativos

Un hombre de 31 años de edad consulta por la presencia de una masa palpable en ¿Qué diagnóstico, entre los siguientes, es el más probable en un niño de 8 años con
el teste derecho, de un mes de evolución, no dolorosa. Su urólogo le realiza una signos inequívocos de pubertad precoz y que, en la exploración, presenta una masa
ecografía testicular, en la que se evidencia una lesión hipoe<oica, bien delimitada, en cl testículo derecho de 2 cm de diilmetrol
intratesticular. los marcadores tumorales a-fetoproteina y �HCG son negativos. la
actitud más correcta de, entre las siguientes, sería: 1 ) Tumor de células de leydig.
2) Scminoma.
1) Dado que los marcadores tumorales son negativos, se descarta neoplasia testicular 3) Tumor del saco vitelina.
y requiere observación. 4) Teraloma.
2) Repetir la ecografía testicular en un plazo de tres meses. 5) Coriocarcinoma.
3) Realización de una tomografía axial compularizada loraco-abdórnino-pélvica.
4) Biopsia transeseralal dcl tcstículo. MIR 03-04, 1 45; Re 4
5) Orquiectomía radical y esperar resultado del patólogo.

MIR 05-06. 107; Re: 5

35
08.
TRAS P LANTE RENAL

Orlentaclon

MIR
lo más imJX)rtante de
este tema son los tipos de OJ las causas más frecuentes de insuficiencia renal crónica son la diabetes mellitus y las glomerulonefritis.
rechazo, que se solap...n con
Inmunologla. Se debe insislir
en el rechazo agudo, que o la clínica característica del rechazo agudo es: fiebre, hipertensión y dolor en el área del injerto.
conviene repasar con las
preguntas de años anleriOfe5. GJ El rechazo agudo produce oliguria, no poliuria.

8 . 1 . I nd i caciones

las dos enfermedades que más comúnmente abocan a una insuficiencia renal terminal irreversible, tratable
mediante un trasplante renal, son la glomerulonefritis y la diabetes mellitus insuli nodependiente.

Otras causas importantes son:


Poliquistosis renal.
Nefroesclerosis hipertensiva.
Enfermedad de Alport.
Nefropatía IgA.
lupus eritematoso sistémico.
Nefroesclerosis.
Nefritis intersticial.
Pielonefritis.
Uropatía obstructiva.

los mejores receptores son individuos jóvenes cuyo fallo renal no se deba a una enfermedad sistémica que pue­
da dañar el riñón trasplantado o causar la muerte por causas extrarrenales. Generalmente se suele mantener al
receptor en tratamiento con diálisis durante un cierto tiempo previo al trasplante.

8.2. Contra i n d icaciones

las contraindicaciones absolutas son las siguientes:


Infección activa.
Enfermedad maligna que no pueda ser erradicada.
Sospecha de no cumplimiento terapéutico del protocolo inmunosupresor.
Glomerulonefritis activa.
Expectativa de vida reducida por enfermedad de base del paciente.
Presencia de anticuerpos preformados frente a antígenos del donante.

7 Preguntas
En referencia a las contraindicaciones relativas del trasplante renal se debe decir que éstas se han ido modifi­
- MIR 02-03, 178
cando a lo largo de los años, al mejorar la técnica y los cuidados prequirúrgicos y postquirúrgicos. En muchas
- MIR 99-00F, 1 4 1 ocasiones, el trasplante plantea menos riesgo que una hemodiálisis crónica.

36
urOlogía �
Actualmente se consideran contraindicaciones relativas la edad avan­ Complicaciones técnicas. Complicaciones vasculares, hemorragia,
zada, la oxalosis, la amiloidosis, la enfermedad iliofemoral oclusiva, hipertensión por estenosis de la arteria renal, trombosis venosa,
las anomalías del tracto urinario inferior O las alteraciones psiquiátricas complicaciones del tracto urinario, necrosis tubular aguda, linfo­
graves (MIR 99-00F, 1 4 1 ). celes.
Complicaciones no técnicas. Infecciones bacterianas y oportunis­
tas en relación con la inmunosupresión, hiperglucemias, compli­
caciones gastrointestinales, h i perparatiroidismo y tumores (cáncer
8.3. Com p l i caciones de piel y de labios, carcinoma in silU de cérvix, l infomas no
Hodgkin; guardan relación con el tratamiento inmunosupresor).
Puede aparecer h ipertensión debida a enfermedad en los riñones
Las complicaciones que se pueden presentar son las siguientes: originales, como consecuencia de rechazo, por estenosis de l a
Rechazo (Tabla 1 8) (MIR 02-03, 1 78). anastomosis d e l a arteria renal o por toxicidad renal por ciclos­
Recurrencia de la enfermedad en el riñón trasplantado. porina.

RECHAZO INICIO PATOGENIA PA TRATAMIENTO

Minutos. días · Ac. preformados · Trombosis microvascular · Nefrectomía del injerto


· CID · Isquemia o infarto
HIpoIagudD · Act. del complemento · PMN en capilares
· Daño endoletelial

Días · Celular (+Ac) · Vasculitis necrotizante · Bolos de esteroides


-- · Respuesta 2.� a Ag-HLA · Ac monoclonales

Semanas · Celular (+Ac) · Forma vascular: mediada por Ac · Bolos de esteroides (la vascular suele ser resistente)
Agudo · Infiltrado de linfocitos · Forma celular: tubulointersticial · Ac monoclonales

Meses. años · Humoral y celular · Intima arterial aumentada · No hay; control de HTA
CnInk:o · Atrofia tubular
· Glomerulopatía

Tabla 18. Rechazo en el trasplante renal

Un paciente de 35 años, con insuficiencia renal crónica, secundaria a pielonefritis 1) Crisis hipertensiva.
crónica recibe un trasplante renal de cadáver con el que comparlía dos idenlidades 2) Infección respiratoria.
en A y 8 Y una en DR. Recibe tratamiento inmunosupresor con cidosporina A y cor­ 3) Pielonefritis aguda del injerto renal.
ticoides en dosis estándar. En el posoperatorio inmediato se observa buena diuresis, y 4) Recidiva de su enfermedad renal.
no es necesario el tratamiento sustitutivo con hemodiálisis. En el 5." día de evolución, 5) Rechazo agudo del injerto renal.
el paciente pt"esenta fiebre de 38", lA de 180/110, oliguria y disminución en la con­
centración urinaria de sodio. El diagnóstico más probable sería: MJR 02-03, 1 78; RC: 5

37
09.
UROPATíA OBSTRUCTIVA

Oflentaclon

MIR
Tema poco preguntado hasta
la fecha. Se debe tener una
GJ la uropatía obstructiva puede producir insuficiencia renal si no se resuelve a tiempo.

idea general y aprender los


Aspectos esenciales. (TI El dolor suele estar presente en la obstrucción aguda. Sin embargo, en la crónica, es frecuente su ausencia.

(l) Después de resolver una uropatía obstructiva puede producirse una fase de poliuria.

9 . 1 . Características

Detención del flujo d e orina e n cualquier punto entre los cálices renales y e l exterior (Tabla 1 9).
Su i mportancia reside en el desarrollo potencial de insuficiencia renal, por lo que tiene importancia la obs­
trucción urinaria bilateral o la unilate-

• RECUERDA
ral sobre riñón único funcionante.
Una obstrucción de más de un mes
la uropatía obstructiva puede producir glomerulonefrltis focal y
de duración puede dar lugar a u n segmentarla.
daño renal funcional y estructural
permanente.

EXTRAPARIETAL EXTRAPARIETAl
INTRAlUMINAl INTRAPARIETAl
(compreslon extnnseca) (dlsfunclon neuromuscular)

litiasis Estenosis congénita Uréter retrocavo Vejiga neur6gena


Tumores (hipernefroma, Estenosis postinfecciosa RiMn en herradura Vejiga automática: lesión sobre
uroteliomal Estenosis postraumática Fibrosis retroperitoneal L1-L3
Necrosis papilar Estenosis isquémica . Tumores (próstata, Vejiga átona: lesión bajo ll-L3
Coágulos vejiga, ginecológicos) Disfunción de la unión
Hiperplasia prostática pieloureteral
ligadura iatr6gena Reflujo vesicoureteral
de uréteres

Tabla 19. Clasificación de la uropatfa obstructiva

9.2. C l ín ica (MIR 98-99F, 1 38)

La forma de presentación depende de los siguientes factores:


Etiología de la obstrucción. Presenta la clínica propia de la enfermedad de base.
Tiempo de evolución. La aguda suele cursar con dolor (cólico nefrítico), siendo la crónica más frecuentemen­
te asintomática.
Lugar de obstrucción:
Tracto urinario inferior (uretra y vejiga). Cursa con retraso para iniciar la micción, disminución de fuerza
y del tamaño del chorro, goteo terminal, hematuria, escozor al orinar, orina turbia, retención aguda de
, Preguntd<;
orina o i ncontinencia paradój ica ("micción por rebosamiento").
• MIR 07..Q8, lOS
Tracto urinario superior (uréter y riñón). Estos pacientes presentan dolor en el flanco (riñón y uréter proxi­
. MIR 98-99f, 138 mal), dolor en flanco con irradiación a genitales (uréter medio) o síndrome miccional (uréter terminal) .

38
urologia �
Después de resolverse una obstrucción, sobre todo si es crónica, puede y su reversibil idad. En los casos en que hay destrucción irreversible de
producirse una fase de poliuria. Esto se debe a que, a nivel tubular, la vía urinaria, es necesario realizar una derivación urinaria definitiva
cuando se ralentiza crónicamente el flujo urinario, se genera una in­ (Figura 28).
sensibilidad a la ADH transitoria (diabetes insípida nefrogénica), de ahí
la poliuria.
MANEJO DE LA UROPATIA OBSTRUCTIVA

¡.
9.3. D i a g nóstico INFRAVESICAl SUPRAVESICAl
(globo vesical) (no globo vesical)

El diagnóstico d e seguridad, la valoración d e la evolución, y el pro­


+ t
SONDA VESICAL
nóstico son ecográficos. Además, son útiles la anamnesis y la explo­
O ClSTOSTOMIA
ración física, la radiología simple (conveniente en litiasis radioopaca),
urografía i ntravenosa (confirma una posible anomalía funcional y útil
en litiasis radiotransparente), cistografía, estudio metabólico (útil en

A
No dilatación Dilatación unilateral Dilatación
prevención de recidivas), Te abdominal, ecografía transrectal, biopsia bilateral

j
prostática dirigida, pielografía retrógrada, nefrostografía, cistoscopia,
Estudio médico
flujometría, cistomanometría y citología urinaria (MIR 07-08, 105).
Birreno Monorreno

Estudio (UIV, TC)


9.4. Trata m iento Fracaso renal

Derivación de vía
Es necesario restablecer el flujo urinario. La mayor parte de las veces urinaria superior
(nefrostomfa/doble J)
se realiza mediante litotricia o corrección quirúrgica. Si la obstrucción
es aguda y/o bilateral, la desobstrucción es urgente, y puede lograrse
mediante un sondaje vesical, talla vesical, catéter ureteral o nefrosto­
Figura 28. Procedim;ento de actuación frente a la uropatla obstructiva
mía. En caso contrario, hay que valorar el grado de sufrimiento renal

39
1 0.
DISF UNCiÓN ERÉCTIL

Orl(>ntaclon

MIR
Tema de reciente
introducción, sencillo
(jJ la causa más frecuente de disfunción eréctil es vascular.
y muy rentable. Se debe
incidir en los factores de o la enfermedad endocrina más relacionada con ella es la diabetes mellitus.
riesgo, el tratamiento El sildenafilo está contraindicado en pacientes que toman nitratos o fármacos donadores de óxido nítrico, en
y sus contraindicaciones. lIJ
pacientes con infarto agudo de miocardio (lAM) en los últimos seis meses, y en pacientes con insuficiencia
Es conveniente conocer muy
bien las preguntas aparecidas cardíaca grave o angina inestable.
con anterioridad.

1 0. 1 . I ntrod ucción

La disfunción eréctil (DE) se define como l a incapacidad persistente o recurrente para conseguir o mantener l a sufi­
ciente rigidez del pene que permita una relación sexual satisfactoria. Debe tener una duración mínima de tres meses.

1 0.2. Preva lencia

En Estados Unidos, en u n estudio en varones de entre 40 y 70 años, se estimó que l a prevalencia global era del
52%. La prevalencia en España se estima en 1 .500.000 a 2.000.000 varones, en torno al 1 2, 1 %.

1 0.3. Etiología

Se puede clasificar en (MIR 04-05, 1 08):


Orgánica. Causas vascu lares (las más frecuentes 60-80%), neurológicas ( 1 0-20°/o), hormonales (5-1 0%) o
locales.
Psicógena.
Mixta. En la mayoría de los casos de etiología orgánica se añade un componente psicológico.

1 0.4. Factores de riesgo

Edad: factor independiente.


Diabetes: es la enfermedad endocrina más frecuente asociada a disfunción eréctil. Significa una probabilidad tres
veces superior de presentar DE. Están implicados mecanismos vasculares, neuropáticos y disfunción gonadal.
Enfermedad cardiovascular: cardiopatía, hipertensión arterial, enfermedad vascular periférica y descenso del
7 Preguntas colesterol HDL se han relacionado de manera clara con la d isfunción eréctil.
Tabaquismo: factor independiente.
- MIR 06-07, 97
- MIR 05-06, 108 Secundaria a fármacos: aquellos que causan hiperprolactinemia, que disminuyen los niveles de testosterona,
- MIR 04-05, 108 psicotropos y antihipertensivos.

40
urologia a

Secundaria a consumo de drogas: cocaína, heroína, etc. un inhibidor de la fosfodiesterasa tipo 5 (PDE5). Induce la relaja­
Trastornos afectivos: depresión. ción del músculo liso del cuerpo cavernoso, l iberando óxido nítrico
(NO). El NO liberado por el endotelio vascular y por las terminacio­
nes nerviosas no adrenérgicaslno colinérgicas es el principal neu­
rotransmisor de la erección. Precisa de deseo sexual y estimulación
1 0. 5 . Diag nóstico previa para su efecto.

Las contraindicaciones absolutas de sildenafilo son:


El diagnóstico debe basarse en los siguientes componentes: Administración concomitante con nitratos o fármacos donadores
Historia clínica y sexual: investigar los posibles factores de riesgo de óxido nítrico por el riesgo de hipotensión grave (dinitrato/mo­
implicados. nonitrato de isosorbide, molsidomina, nicorandil, nitroglicerina,
Exploración física: encaminada a descartar enfermedad vascu­ nitroprusiato sódico) (MIR 06-07, 97; MIR 05-06, 108).
lar, enfermedades neurológicas, trastornos genitales y endocri­ Pacientes en los que esté desaconsejada la actividad sexual (an­
nopatías. En varones mayores de 50 años se incluirá tacto rectal. gina inestable, insuficiencia cardíaca o i nfarto reciente, hace
Determinaciones analíticas: glucemia basal, perfil l ipídico, testos­ menos de seis meses).
terona total y l ibre y prolactina. Además, es conveniente solicitar
hemograma, función renal y hepática. Apomorfina: agonista dopaminérgico que actúa a nivel central so­
Pruebas especializadas: únicamente en ocasiones muy seleccionadas. bre el mecanismo de la erección. Está contraindicado en sujetos que
tengan desaconsejada la actividad sexual.

1 0.6. Trata m i e nto Seg unda línea

Se puede estructurar en tres escalones o etapas, que serán superadas en Terapia intracavernosa: alprostadil (PGE1), mediante inyección di­
función de fracaso del escalón previo. recta en 105 cuerpos cavernosos. Otros fármacos son la papaverina
y la fentolamina.

Fármacos orales
Tercera línea

Citrato de si ldenafilo: se considera actualmente como el tratamien­


to farmacológico de elección en la DE. Hoy en día existen nuevos Cirugía de revascularización (venosa, arterial). Implante de prótesis
fármacos basados en él, como vardenafilo y tadalafilo. Se trata de de pene.

Paciente de 6J años, en tratamiento a demanda con citrato de sildenafilo por presen­ J) Digoxina.
tar disfunción eréctil de años de evolución. Señale cuál de los siguientes fármacos 4) lndapamida.
NO asociaría en ningún caso a su tratamiento: 5) Mononitrato de isosorbide.

1 ) Amiodarona. MJR 05-06, 108; RC: 5


2) Verapamilo.

41

También podría gustarte